exam 3

Réussis tes devoirs et examens dès maintenant avec Quizwiz!

A client reports occasionally experiencing hoarseness. In response to this statement, the nurse asks, "What makes the hoarseness go away?" Which characteristic of the client's symptom is the nurse assessing?

relieving factors

Which technique is appropriate for the nurse to use to palpate a client's breast?

palms of both hands The nurse should use the flat pads of three fingers to palpate the breast of the client for accurate assessment. The tips of four fingers, palms of both hands, or palm and fingers of one hand are not used for assessing the breasts as they may not give accurate results on examination.

A nurse palpates a weak left radial artery on a client. What should the nurse do next?

palpate both radial arteries for symmetry Extremities should always be assessed simultaneously for symmetry. If the radial arteries are both weak, this may indicate a problem with peripheral circulation. The nurse should then assess the ulnar artery pulses to determine the presence of arterial insufficiency. The hands should be assessed for pallor and coolness, which would also be present with arterial insufficiency. All findings should be documented in the client's record.

What instruction should the nurse provide to a client to assess for dimpling or retraction of breast tissue?

raise arms over the head To bring out dimpling or retraction that may otherwise be invisible, ask the client to raise the arms over the head. Shrugging the shoulders, bending the arms at the elbows, and extending the arms out to the side are not actions to bring out breast dimpling or retractions.

The roof of the oral cavity of the mouth is formed by the anterior hard palate and the

soft palate.

In order to effectively assess the oral mucosa, the nurse should have which assessment tools available?

tongue depressor gloves penlight

Mrs. Ash, a client in her 50s, has told the nurse during her most recent visit to the clinic that she and her circle of friends have discontinued breast self-examination (BSE) since hearing and reading that the practice is now considered ineffective. How can the nurse best respond to Mrs. Ash?

"BSE is certainly not a replacement for other screening methods, but a high proportion of breast masses are in fact detected by women themselves." While BSE does not reduce breast cancer mortality, the fact remains that a high proportion of breast masses are detected by women themselves. BSE should be coupled with mammography and clinical examination.

The nurse is performing a cardiovascular assessment and wants to identify the client's risk factors for cardiovascular disease. Which questions will help the nurse assess the client's risk? Select all that apply. Hint: Focused Interview Questions "Do you use recreational drugs?" "Tell me about your exercise routine." "Do you know your cholesterol numbers?" "Do you smoke?" "Are you married or do you have a significant other?

"Do you use recreational drugs?" "Tell me about your exercise routine." "Do you know your cholesterol numbers?" "Do you smoke?" The use of recreational drugs, especially cocaine can cause spasm of the coronary arteries leading to myocardial ischemia or injury of the myocardium. The benefits of exercise on the heart are well documented. Elevated cholesterol and triglyceride levels are associated with cardiovascular disease. Smoking has been linked to hypertension; hypertension is closely linked to coronary artery disease. The client's relationship status is not a risk factor for cardiovascular disease.

The client tells the nurse that she has benign breast disease and so she is not worried about any lumps or nodules in her breasts. How would the nurse best respond?

"It is important to perform self breast examinations as there could be changes or additional lumps in your breasts that would need further examination."

The parents of a newborn appear anxious and express concern upon seeing the pediatrician palpate and gently express a small amount of whitish, milk-like liquid from the baby's nipples. What is the nurse's best response?

"Nothing is wrong with your baby. This liquid will clear up spontaneously." Enlarged breast tissue and white discharge (commonly called "witch's milk") in newborns of either gender may occur for the first few weeks of life, secondary to the effects of maternal estrogens (Colvin and Abdullatif, 2013). If breast enlargement, witch's milk, or both are present, it is important to reassure the newborn's parents/caregivers that nothing is wrong and the conditions will resolve spontaneously. The baby does not need further evaluation, hormone therapy, or a change in diet. This condition is considered normal for the first few weeks.

A client has been found to have a breast lump and an ultrasound has been ordered. The client voices concerns to the nurse she is afraid of the painful testing she is going to endure. How should the nurse best respond?

"This noninvasive test uses high frequency waves to determine if the mass is solid or cystic." An ultrasound is a noninvasive test that uses sound waves. An MRI uses a magnetic field and mammography uses x-ray. A fine needle biopsy is an invasive procedure.

An older female client states she is upset about her sagging breasts. She states, "I know it's because I haven't been sexually active since my husband died and my hormones are all dried up." What is the nurse's best response?

"What you're experiencing is an expected effect of the aging process."' The suspensory ligaments relax, causing breasts to sag and droop. Breasts also decrease in size and lose elasticity (Hassiotou & Geddes, 2013). Nipples become smaller, flatter, and less erectile. Axillary hair may stop growing at this time. These changes are more apparent in the eighth and ninth decades of life. The client's concern is the physical change of sagging breasts, not meeting men. Sagging breasts are expected changes; hormone therapy is not recommended. Appearance may be very signigicant to some clients.

Upon assessment of the tonsils, the nurse finds them to be obstructing 30% of midline. This nurse would document this as what?

1+ When tonsils obstruct 25-50% of midline, it would be documented as 2+. A 1+ is an obstruction of up to 25% of midline. A 3+ is an obstruction of 50-75% of midline. If the tonsils obstruct 75-100% of midline, it is a 4+.

A nurse examines a client with complaints of a sore throat and finds that the tonsils are just visible. Using a grading scale of 1+ to 4+, how should the nurse appropriately document the tonsils?

1+ The nurse should document the tonsillar grading as 1+ because the tonsils are just visible. Grade 2 tonsils are midway between the tonsillar pillars and the uvula. Tonsils that touch the uvula are graded 3+, and tonsils that are so enlarged that they touch each other are graded 4+.

Which of the following wounds is most likely attributable to neuropathy?

A painless wound on the sole of the client's foot, which is surrounded by calloused skin Neuropathic ulcers tend to develop on pressure points, such as the sole of the foot, and are often free of pain. Painful wounds surrounded by healthy skin are associated with arterial insufficiency and moderately painful ankle wounds surrounded by pigmented skin are often associated with venous ulcers.

A 70-year-old patient is scheduled for open-heart surgery. The surgeon plans to use the great saphenous vein for the coronary bypass grafts. The patient asks, "What happens to my circulation when the veins are removed?" The nurse should reply:

ANS: "Because the deeper veins in your leg are in good condition, this vein can be removed without harming your circulation." As long as the femoral and popliteal veins remain intact, the superficial veins can be excised without harming the circulation. The other responses are not correct.

The nurse is describing a weak, thready pulse on the documentation flow sheet. Which statement is correct?

ANS: "Hard to palpate, may fade in and out, easily obliterated by pressure." A weak, thready pulse is hard to palpate, may fade in and out, and is easily obliterated by pressure. It is associated with decreased cardiac output and peripheral arterial disease.

The nurse is teaching a review class on the lymphatic system. A participant shows correct understanding of the material with which statement?

ANS: "The flow of lymph is slow compared with that of the blood." The flow of lymph is slow compared with that of the blood. Lymph flow is not propelled by the heart, but rather by contracting skeletal muscles, pressure changes secondary to breathing, and by contraction of the vessel walls. Lymph does not absorb lipids from the biliary tract. The vessels do have valves, so flow is one way from the tissue spaces to the bloodstream.

During an assessment the nurse has elevated a patient's legs 12 inches off the table and has had him wag his feet to drain off venous blood. After helping him to sit up and dangle his legs over the side of the table, the nurse should expect a normal finding at this point would be:

ANS: venous filling within 15 seconds. In this test it normally takes 10 seconds or less for the color to return to the feet and 15 seconds for the veins of the feet to fill. Marked elevational pallor as well as delayed venous filling occurs with arterial insufficiency.

The nurse assesses the frontal sinus where?

Above the eyes

A 23-year-old computer programmer comes to the office for an annual examination. She has recently become sexually active and wants to start oral contraception. Her only complaint is that the skin in her armpits has become darker. She states it looks like dirt; she scrubs her skin nightly with soap and water, but the colour stays. Her past medical symptoms consist of acne and mild obesity. Her periods have been irregular for 3 years. Her mother has type 2 diabetes and her father has high blood pressure. The client denies tobacco but drinks four to five drinks on Friday and Saturday nights. She denies any illegal drug use. Examination shows a mildly obese woman breathing comfortably. Her vital signs are unremarkable. Inspection of the axillae reveals dark velvet-like skin. Her annual examination is otherwise unremarkable. What disorder of the breast or axilla is she most likely to have?

Acanthosis nigricans

The nurse is preparing an educational session for a group of older adults regarding risk factors for cardiovascular disease. Which group would the nurse note as having the highest incidence of hypertension? Hint: Cultural and Environmental Considerations African Americans Asians Whites Native Americans

African Americans Hypertension is a risk factor for cardiovascular disease. Hypertension occurs more frequently in African Americans and Hispanics than in other groups. Hypertension does not occur as frequently in Asians, Caucasians, and Native Americans.

The Kiesselbach plexus is the most common site for what?

Anterior nosebleeds

The nurse is auscultating across the precordial surface and is focusing on the second heart sound (S2). Which heart valves produce this sound? Hint: Anatomy and Physiology Review, Heart Sounds Mitral and tricuspid Aortic and pulmonic Mitral and pulmonic Aortic and tricuspid

Aortic and pulmonic The second heart sound (S2) is produced by the closure of the aortic and pulmonic valves. Closure of the aortic and pulmonic valves produces the first heart sound. Closure of the mitral and tricuspid valves produces the first heart sound. The mitral and pulmonic valves do not close at the same time to produce a heart sound. The aortic and tricuspid valves do not close at the same time to produce a heart sound.

Which action by the nurse is appropriate to provide a clear view of the uvula for observation?

Ask the client to say "aaah" Asking the client to say "aaah" and instructing him or her to open the mouth wide makes the uvula more clear for observation. The nurse should depress the client's tongue slightly off center to prevent the gag reflex during observation of the uvula. Depressing the back of the tongue would elicit the gag reflex. Having the client stick out the tongue would not provide a clear view of the uvula.

A nurse is inspecting a client's breasts. The nurse notices that one breast is larger than the other. Which action should the nurse take next?

Ask the client whether the larger breast has increased in size recently Breasts can be a variety of sizes and are somewhat round and pendulous. One breast may normally be larger than the other. However, a recent increase in the size of one breast may indicate inflammation or an abnormal growth; it thus would be best for the nurse to inquire about any recent changes in breast size. There is no need to inform the physician immediately. Inquiring about the client's family history of breast cancer should have occurred earlier, during the interview.

The nurse identifies this as trapping debris and propelling it toward the nasopharynx.

Cilia

Which food is most appropriate for the nurse to recommend for a client who suffers frequent nosebleeds due to hereditary hemorrhagic telangiectasia?

Cooked vegetables Counsel HHT patients regarding lifestyle and dietary factors that may potentially decrease the occurrence of nosebleeds, such as room humidification and the use of topical nasal creams along with nasal saline treatments. Dietary recommendations including decreasing foods high in salicylates such as red wine, spices, chocolate, coffee, and some fruits. Provide education about supplements with antiplatelet activity, such as garlic, ginger, ginseng, gingko, and vitamin E (Silva et al., 2013).

A nurse is working with a client who has an impaired ability to move the tongue. He explains that he was in an automobile accident many years ago and suffered nerve damage that resulted in this condition. Which nerve should the nurse suspect was damaged in this client?

Cranial nerve XII (hypoglossal)

A hospitalized client continues to exhibit residual effects of a stroke. Which symptom is the priority concern?

Dysphagia

During assessment of the oral cavity, the nurse examines the salivary glands. Which area of the mouth should the nurse assess to inspect for the Wharton's ducts?

Either side of the frenulum on the floor of the mouth

A nurse prepares an educational session on the importance of oral hygiene. Which teaching tip should be included in the section about preventing gingivitis? You Selected:

Ensure that dental restorations fit well. Poorly fitting dental restorations or loose fitting dentures may contribute to inflammation of the gums. Prevention of gingivitis includes ensuring plaque does not build up on the gumline, so clients should be encouraged to brush and floss at least twice per day. More optimally, clients should be brushing and flossing after each meal. A firm toothbrush can cause or aggravate inflammation of the gums. A soft toothbrush is recommended in the prevention and management of gingivitis. Flossing should be done regularly even if there is bleeding at the gums. The aim is to prevent a build up of plaque and remove food residue to prevent infection.

The frontal sinuses are the only ones readily accessible to clinical examination.

False

A child presents to the health care facility with new onset of a foul-smelling, purulent drainage from the right nare. The mother states that no other signs of an upper respiratory tract infection are present. What is an appropriate action by the nurse?

Inspect the nostrils with an otoscope Because the drainage is unilateral, the most likely cause is a foreign body obstruction. The nurse should inspect the nostrils for patency and the presence of a foreign body. It is not a normal finding in children to have unilateral foul-smelling drainage from the nose. This child will not need an antibiotic, thus the nurse does not need to assess for allergies to medication. Blowing the nose may or may not dislodge the object and may cause further trauma to the nare.

The client is experiencing red gums that are bleeding. The nurse identifies this as what?

Gingivitis Gingivitis is an inflammation of the gums with bleeding. Torous palatinus is a bony risk running in the middle of the hard palate. Bidfid uvula is a minor cleft. Ludwig's angina is a swelling that pushes the tongue up and back.

During the physical examination of the mouth, the nurse identifies vesicular eruptions along the patient's lips and surrounding skin. The nurse would document this finding as being:

Herpes simplex

A staff educator from the hospital is providing an event for the hospital staff. The educator is talking about health promotion activities for people with diseases of the nose, mouth, throat, and sinuses. What would the educator include in the presentation?

How to reduce periodontal disease Major risk reduction and health promotion goals in assessment of the nose, sinuses, mouth, and throat are related to various issues, including tobacco use, obstructive sleep apnea, oral health, and cancer. Health goals include reducing periodontal disease.

A 57-year-old maintenance worker comes to the office for evaluation of pain in his legs. He is a two-pack per day smoker since the age of 16, but he is otherwise healthy. The nurse is concerned that the client may have peripheral vascular disease. Which of the following is part of common or concerning symptoms for the peripheral vascular system?

Intermittent claudication is leg pain that occurs with walking and is relieved by rest. It is a key symptom of peripheral vascular disease. This symptom is present in only about one third of clients with significant arterial disease and, if found, calls for more aggressive management of cardiovascular risk factors. Screening with ankle-brachial index can help detect this problem.

An emergency department nurse is caring for a young child with intractable nose bleeds. What is the most common site of epistaxis?

Kiesselbach plexus

A client visits the clinic and tells the nurse that she had a mastectomy 2 years ago. The nurse should assess the client for

Lymphedema results from blocked lymphatic circulation, which may be caused by breast surgery. It usually affects one extremity, causing induration and nonpitting edema.

An older adult client who wears dentures reports having soreness of the gums. Which intervention should the nurse recommend to the client to alleviate this problem?

Massage the gums daily. Regular massage of the gums relieves soreness and pressure from dentures on the underlying soft tissue. Avoiding excessive intake of sugary foods is a strategy to reduce the risk of cariogenic bacteria. The client with dentures will not use toothpaste but rather a cleanser specific for dentures. The client with dentures should have a dental examination every year; however, this intervention will not provide immediate relief from the gum soreness.

A nurse inspects the lower extremities of a client and notices that the legs appear asymmetric. What should the nurse do first in regards to this finding?

Measure the diameter of the calves The nurse should complete the inspection process before going on to the other physical assessment techniques. After inspecting asymmetry of the legs, the nurse should measure the calves to determine the exact difference in diameter. Then the nurse can palpate for edema and temperature and notify the health care provider with the information once it is all gathered.

The nurse is educating a group of women about heart attacks and explains that women may experience symptoms differently than men. Which symptoms should the nurse highlight as ones that tend to be more common in women having heart attacks? Select all that apply. Hint: Questions Related to Symptoms Chest pain radiating to the jaw Nausea and vomiting Extreme fatigue Diaphoresis Indigestion

Nausea and vomiting Extreme fatigue Indigestion During myocardial infarction (heart attack), women may experience nausea and vomiting, indigestion, or extreme fatigue in the absence of chest pain. Men often experience prolonged, dull chest pain radiating to the jaw or shoulder accompanied by diaphoresis, shortness of breath, and nausea. Both men and women may experience shortness of breath.

Which of the following assessment findings is most likely benign on breast examination?

One breast larger than the other Asymmetry in size of the breasts is a common benign finding. The others are concerning for underlying malignancy.

What is the common channel for the respiratory and digestive systems?

Oropharynx

A client presents to the health care clinic complaining of a sore throat. In examining the client's mouth and throat, the nurse notices that the tonsils on both sides of the oropharynx at the end of the soft palate are swollen. Which tonsils are these?

Palatine Masses of lymphoid tissue referred to as the palatine tonsils are located on both sides of the oropharynx at the end of the soft palate between the anterior and posterior pillars. The lingual tonsils lie at the base of the tongue. Pharyngeal tonsils or adenoids are found high in the nasopharynx. Paranasal refers to sinuses, not tonsils.

A nurse is examining the breasts of a woman who has had a mastectomy. Which of the following should the nurse do?

Palpate the scar for redness, lesions, lumps, swelling, or tenderness If the client has had a mastectomy or lumpectomy, it is still important to perform a thorough examination. Palpate the scar and any remaining breast or axillary tissue for redness, lesions, lumps, swelling, or tenderness. White scar tissue in a client who underwent a mastectomy or lumpectomy is a normal finding and need not be referred.

A nurse is discussing breast self-examination (BSE) with a 60-year-old woman. Which of the following should the nurse recommend?

Picking a set day of the month that the client will remember on which to perform BSE Older clients and others who no longer menstruate may find it helpful to pick a set day of the month for BSE, a date that they will remember each month such as the day of the month they were born. Although BSE is not required, the nurse should not encourage the client to discontinue it if she is already performing it. It is unlikely that the client is still menstruating at her age. The BSE, if performed, should be done monthly, not annually.

The nurse observes a white patchy area in the pharyngeal fossa of a client. What is the nurse's best action?

Prepare client for a biopsy of the lesion. The phayngeal fossa is the most common site of oral cancer. A whitish area is a suspicous finding and will likely be biopsied. Gargling with saline and antibiotics are not recommended. This finding does not indicate a need for a tonsillectomy. Indications for tonsillectomy are repeated tonsillitis and/or tonsil hypertrophy.

A nonpregnant female presents to the health care facility and reports new onset of breast discharge. The nurse assesses the discharge to be milky in appearance without breast tenderness or masses. What additional data should the nurse obtain from this client?

Prescribed medications such as antipsychotic agents A persistent milk secretion (galactorrhea) from the breasts in a nonpregnant, nonlactating woman can be caused by the intake of hormones, contraceptives, and some antipsychotic agents, such as haloperidol (Haldol). Recent surgeries or trauma and exposure to chemicals are not known to cause persistent milk discharge. Excessive alcohol intake is a risk factor for the development of breast cancer.

A nurse, explaining the cardiac circulation to cardiac rehabilitation clients, wants to include the oxygenation of the heart muscle. Which structure carries deoxygenated blood to the lungs? Hint: Anatomy and Physiology Review, Pulmonary Circulation Pulmonary artery Pulmonary vein Great cardiac vein Anterior cardiac vein

Pulmonary artery The pulmonary artery is the only artery in the body to carry deoxygenated blood. Deoxygenated blood flows into the right atrium. It is ejected through the tricuspid valve into the right ventricle and passes through the pulmonic valve into the pulmonary artery and pulmonary circulation. Oxygenated blood is then returned to the left atrium via the pulmonary veins, which are the only veins to carry oxygenated blood. The great cardiac vein carries deoxygenated blood to the coronary sinus. The anterior cardiac vein carries deoxygenated blood to the great cardiac vein, which then empties into the coronary sinus.

During an examination of the oral cavity, which technique by the nurse is appropriate to examine the gums and teeth?

Put on gloves and retract the client's lips and cheeks.

A client seeks medical attention for the condition shown. What finding does the nurse anticipate? (white fingers)

Raynaud's disease is a vascular disorder caused by vasoconstriction or vasospasm of the fingers or toes, characterized by rapid changes of color (pallor, cyanosis, and redness), swelling, pain, numbness, tingling, burning, throbbing, and coldness. The disorder commonly occurs bilaterally; symptoms last minutes to hours. Venous insufficiency, deep vein thrombosis, and arterial insufficiency all affect the blood vessels of the lower extremities.

A nurse is assessing the mouth of an older client. Which of the following findings is common among older adults?

Receding and ischemic gums The gums recede, become ischemic, and undergo fibrotic changes as a person ages. A bifid uvula is a common finding in Native Americans, not among older adults. Brown spots on the chewing surface of teeth is an indication of tooth decay and is not associated with aging per se, nor are enlarged palatine tonsils, which are an indicator of tonsillitis.

A nurse is examining the nose of a client diagnosed with an upper respiratory tract infection. Which characteristics of the nasal mucosa should the nurse expect to find during assessment of a client with an upper respiratory tract infection?

Red, swollen, with purulent discharge The nurse should find red, swollen nasal mucosa with purulent discharge in the client diagnosed with upper respiratory tract infection. Dark pink, moist nasal mucosa which is free of exudate is a normal finding. Pale pink, swollen nasal mucosa with watery exudate and bluish gray, swollen nasal mucosa with watery exudate is found in cases of allergy.

The nurse is planning instructions for a patient with a broken nose. What teaching will be included to address the alterations in nasal function? (Select all that apply.)

Safety measures because of a loss of smell Importance to increase oral fluids How to breathe through the mouth Remind that the voice may sound different

When doing a shift assessment on a newly admitted client, the nurse notes lack of hair on the right lower extremity; thickened nails on the right lower digits; dry, flaky skin on the right lower extremity; and diminished tibial pulses bilaterally and absent pedal pulses. What nursing diagnosis should this client receive?

Signs of altered tissue perfusion, arterial related to reduced blood flow include decreased oxygen, resulting in a failure to nourish tissues at the capillary level; reduced hair on the extremity; thick nails; dry skin; weak or absent pulses; pale skin; cool, reduced sensation; and prolonged capillary refill. The other options are distracters to the question.

The nurse is performing the assessment shown. What is the nurse assessing in this client?

Sinuses Transillumination is a traditional method by which clinicians assess sinus cavities. This method has limited clinical significance and provides inconsistent results. This technique is not used to assess the nasal turbinates, the septum, or the sense of smell.

On examination of a client, the nurse detects a fecal odor to the breath. The nurse recognizes this finding as characteristic of what disease process?

Small bowel obstruction Clients with small bowel obstructions have a fecal smell to their breath. The nurse should suspect the client of having diabetic ketoacidosis if there is a fruity smell on the breath. Clients with end-stage liver disease have a sulfur odor in their breath. Clients with respiratory infection have foul odors in their breath.

The nurse is calculating the cardiac output for a client with a heart rate of 88 beats per minute. What other measurements does the nurse need to complete this calculation? Hint: Special Considerations, Measurements of Cardiac Function Cardiac index Blood pressure Mean arterial pressure Stroke volume

Stroke volume To calculate the cardiac output, the stroke volume (amount of blood that is ejected with each heartbeat) is multiplied by the heart rate. Cardiac output = stroke volume × heart rate for 1 minute. Cardiac index takes into account the client's weight, which is not used in calculating the cardiac output. Blood pressure is not used when calculating the cardiac output. Mean arterial pressure is not used in cardiac output calculation. Mean arterial pressure is the difference between systolic and diastolic blood pressure.

When assessing temperature of the skin, which portion of the hand should the examiner use?

The backs of the fingers are thought to be the most temperature sensitive, perhaps because the skin is thinnest there. The nurse may have difficulty detecting subtle differences without using the backs of the fingers.

During the cardiac assessment, the nurse finds that a client has jugular vein distention. What does this finding suggest? Hint: Techniques and Normal Findings, Inspect the Jugular Veins The client could have fluid overload. The client is dehydrated. The client has an infection. This is an expected finding for the jugular veins.

The client could have fluid overload. Distention of the neck veins indicates elevated central venous pressure commonly seen with heart failure, fluid overload, or pressure on the superior vena cava. With dehydration, the neck veins would be flat, not distended. Infection would not impact the jugular veins. Distention of the jugular veins is not a normal finding.

During the examination of an 8-month-pregnant client, the nurse measures the blood pressure at 160/98 and notes bilateral edema of the ankles. What do these findings suggest to the nurse? Hint: Special Considerations, The Pregnant Female This is a normal finding during the eighth month of pregnancy. The client may have a prolapsed mitral valve. The client may go into early labor. The client may be exhibiting signs of preeclampsia.

The client may be exhibiting signs of preeclampsia. Hypertension and edema are symptoms of preeclampsia and can place the mother and infant at risk if not treated. This is not a normal finding at any stage of pregnancy. Elevated blood pressure and edema are not related to a prolapsed mitral valve. Hypertension and edema do not indicate early labor.

The lateral lymph nodes are located along the upper humerus and drain most of the arm.

The lateral nodes drain most of the arms, and the central nodes receive drainage from the anterior, posterior, and lateral lymph nodes. A small proportion of the lymph also flows into the infraclavicular or supraclavicular lymph nodes or deeper into nodes within the chest or abdomen.

nurse assessing pulse- posterior tibial

The posterior tibial pulse is located in the groove between the medial malleolus and Achilles tendon. The femoral pulse is about halfway between the symphysis pubis and anterior iliac spine, just below the inguinal ligament. The popliteal pulse is located behind the knee lateral to the medial tendon. The dorsalis pedis pulse is located halfway up the foot, immediately lateral to the extensor tendon of the great toe.

The nurse is palpating the precordial surface of an adult and notes a mild pulsation in the fifth intercostal space at the midclavicular line. What does this finding suggest to the nurse? Hint: Techniques and Normal Findings; Palpation The client may have had a myocardial infarction. This is an abnormal finding related to emphysema. This is a normal finding and the location of the point of maximum impulse (PMI)/apical impulse. The client likely has a heart murmur.

This is a normal finding and the location of the point of maximum impulse (PMI)/apical impulse. The presence of a palpable pulsation in this location identifies the point of maximum impulse (PMI) or apical impulse. This is a normal and expected finding. This doesn't signify that the client has had an MI. It is a normal, not an abnormal, finding and not related to the presence of emphysema. Heart sounds should not be altered based on this normal finding.

During the assessment of the precordial surface of an adult client, the nurse inspects and palpates a heave at the apex of the heart with each heartbeat. What does this finding suggest to the nurse? Hint: Techniques and Normal Findings, Palpation This is a normal finding in an athletic adult. This is an abnormal finding possibly due to left ventricular hypertrophy. The client is likely having a myocardial infarction. The client likely has a history of a heart murmur.

This is an abnormal finding possibly due to left ventricular hypertrophy. The presence of a heave is considered abnormal over any location on the precordial surface. In this case, it may be indicative of hypertrophy of the left ventricle. The presence of a heave is an abnormal finding regardless of the age or athletic condition of the client. The presence of a lift is not associated with a current MI, nor is it indicative of the presence of a heart murmur.

Upon inspection of a client's oral cavity, a nurse observes a bifid uvula. What should the nurse recognize about this finding?

This is often a normal finding in the Native American population

A trauma client reports pain in the left lower extremity. The nurse notes that the extremity has pallor. Pedal pulses are diminished, and paresthesia is present. What nursing diagnosis might the nurse use?

Those with risk for peripheral neurovascular dysfunction are at risk for a disruption in circulation, sensation, or motion of an extremity. Risk factors include trauma, fractures, mechanical compression, surgery, burns, immobilization, and obstruction. The other options are distracters to the question.

When visualizing the structures of the nose, the nurse recalls that air travels from the anterior nares to the trachea through the:

Vestibule, nasal passages, and nasopharynx After entering the anterior nares, air enters the vestibule and passes through the narrow nasal passage to the nasopharynx.

The submandibular glands open under the tongue through openings called

Wharton ducts. The submandibular glands, located in the lower jaw, open under the tongue on either side of the frenulum through openings called Wharton's ducts.

After palpating the radial pulse of an adult client, the nurse suspects arterial insufficiency. The nurse should next assess the client's

You can also palpate the brachial pulses if you suspect arterial insufficiency. Do this by placing the first three fingertips of each hand at the client's right and left medial antecubital creases. Alternatively, palpate the brachial pulse in the groove between the biceps and triceps.

A client has been diagnosed with a myocardial infarction that has damaged a part of the right atrium. The nurse monitors this client for: Hint: Anatomy and Physiology Review, Cardiac Conduction System shortness of breath increase in collateral circulation onset of aortic regurgitation a change in the rate and rhythm of the client's heartbeat

a change in the rate and rhythm of the client's heartbeat The sinoatrial node, which initiates the electrical impulse, is called the pacemaker of the heart and has a rate of 60-100 per minute. This node is located at the junction of the superior vena cava and the right atrium. Shortness of breath would be seen with left-sided heart failure. A myocardial infarction would not cause an increase in the collateral circulation. Aortic regurgitation would be seen with abnormalities of the aortic valve, which is located in the left side of the heart, not the right atrium.

During the admission assessment, the nurse identifies the client has a history of Raynaud's. What assessment finding would the nurse expect to find?

cold fingers and hands Raynaud's is exhibited by cold fingers and hands. Cool extremities could be due to a cool room or arterial insufficiency. A capillary refill of less than 2 seconds is normal.

Fibrous tissue that provides support for the glandular tissue of the breasts is termed

cooper ligaments The fibrous tissue provides support for the glandular tissue largely by way of bands called Cooper's ligaments (suspensory ligaments). These ligaments run from the skin through the breast and attach to the deep fascia of the muscles of the anterior chest wall.

The largest arteries of the upper extremities are the

brachial arteries

The client is experiencing septic shock. What assessment finding would the nurse expect to find?

capillary refill greater than 2 seconds The client experiencing septic shock would have a capillary refill greater than 2 seconds. The temperature may or many not be normal, blood pressure would be low and extremities would be cool.

A patient has developed an infection of the right forearm. The nurse will focus the assessment of the patient's lymphatic system on which area?

epitrochlear Lymphatics from the ulnar surface of the forearm drain first into the epitrochlear nodes, which are located on the medial surface of the arm approximately 3 cm above the elbow. Lymphatics from the rest of the arm drain into the lateral and central axillary nodes and a few may drain directly into the infraclavicular nodes.

The nurse is unable to palpate a client's left popliteal artery. Which artery should be assessed to determine the presence of blood flow in the left leg?

femoral artery Since the nurse is unable to palpate the popliteal artery, the femoral artery should be palpated to determine if there is blood flow in the extremity. The dorsalis pedis and posterior tibial arteries are located in the foot. If the popliteal artery cannot be felt, it is likely that these two arteries will not be palpable either. Saphenous is a vein and is not routinely palpated to determine blood flow in an extremity.

Benign conditions of the breast include

fibrocystic changes Cysts (due to BBD) are common lumps that are usually elliptical or round, soft, and mobile. Size may vary, and they often occur in multiple numbers, usually in both breasts, and frequently in the upper outer quadrants.

A nurse receives an order to perform a compression test to assess the competence of the valves in a client's varicose veins. Which action by the nurse demonstrates the correct way to perform this test?

firmly compress the lower portion of the varicose vein The nurse should firmly compress the lower portion of the varicose vein with one hand. The nurse should ask the client to stand, not sit, on a chair for the examination. The second hand should be placed 6 to 8 inches, not 3 to 4 inches, above the first hand. The nurse should feel for a pulsation to the fingers in the upper hand.

A middle-aged female tells the nurse that she is concerned because her breasts are not as firm as they used to be. What is an appropriate response by the nurse?

firmness of the breasts decrease with lower estrogen levels

During the examination of the breasts, a young adult patient's nipples becomes erect and the areolas pucker. The nurse explains to the patient that these findings indicate:

intact tactile stimulation of the area With tactile stimulation of the breast, the nipple will become more erect and the areola will pucker. This is a normal smooth muscle response. This response does not mean that the patient will have difficulty with breast-feeding. This response does not mean that the patient has an overabundance of areolar glands.

The lymph nodes that are responsible for drainage from the arms are the

lateral lymph nodes

The nurse is preparing to examine the sinuses of an adult client. After examining the frontal sinuses, the nurse should proceed to examine the

maxillary sinuses.

When performing a breast assessment on a clinic patient, the nurse practitioner notes scaly lesions that begin at the nipple and move toward a lump behind the nipple well. The NP would know that further assessment for what would be necessary?

paget's disease Paget's disease produces scaly lesions that begin at the nipple and progress to a lump behind the nipple well. Severe pain (mastalgia) is more likely to result from trauma or infection. Single breast masses can indicate benign conditions (eg, cysts, fibroadenoma, fat necrosis, lipoma) or more serious conditions (eg, cancer).

At puberty, the female breasts enlarge in response to estrogen and

progesterone

What should a nurse do if a posterior tibial pulse cannot be obtained on a client with edema of the feet?

use a doppler to evaluate presence of a pulse Edema in the feet or ankles may make it difficult or impossible to palpate for the posterior tibial pulse. In these cases, a Doppler should be used to assess for adequate circulation. Elevating the feet will not enhance the pulse. Assessing temperature is not an alternative for assessing circulation because this client demonstrates edema, which shows that circulation is compromised. About 15% of healthy clients may not have a posterior tibial pulse present.

A client has a brownish discoloration of the skin of both lower legs. What should the nurse suspect is occurring with this client?

venous insufficiency Brownish discoloration just above the malleolus suggests chronic venous insufficiency. There are no specific skin changes associated with atherosclerosis. The lower extremities in the dependent position would be pale in color in arterial insufficiency. The extremity would be warm and edematous with a deep vein thrombosis.

During a physical examination, the nurse detects warm skin and brown pigmentation around an adult client's ankles. The nurse suspects that the client may be experiencing

venous insufficiency Manifestations of venous insufficiency include cramping pain, thickened tough skin, and areas of hyperpigmentation around the medial and lateral malleolus.

When you enter the room of a hospitalized patient, you note that the patient is guarding her left leg, which is swollen and reddened. You should identify the signs and symptoms of what complication of hospitalization?

venous thromboembolism Edema, pain or achiness, erythema, and warmth in the leg are common signs and symptoms of venous thromboembolism.

What pattern of palpation is currently the best validated technique for detecting breast masses?

vertical strip pattern Although a circular or wedge pattern can be used, the vertical strip pattern is currently the best validated technique for detecting breast masses.

A client presents to the health care clinic with reports of inability to concentrate at work and daily frontal headaches for the past two weeks. What additional information should the nurse ask this client?

"Are you experiencing sinus pressure and congestion?' The client has a recent onset of a frontal headache and the nurse should collect information on additional findings of a sinus infection. Family history of headaches will not provide information about the current headache. High blood pressure causes a headache in the occipital area. A previous injury will not explain the recent acute onset of headache that the client is now experiencing.

A nurse is assessing a small child who has lead poisoning. Which characteristic of the gums should the nurse expect this client?

A grey-white line

A nurse finds crepitus when palpating over a client's maxillary sinuses. Which of the following should the nurse most suspect in this client?

A large amount of exudate in the sinuses Frontal or maxillary sinuses are tender to palpation in clients with allergies or acute bacterial rhinosinusitis. If the client has a large amount of exudate, you may feel crepitus upon palpation over the maxillary sinuses. Normal, air-filled sinuses would not demonstrate crepitus. Obstruction of the nostril by a foreign object would prevent sniffing or blowing air through the nostrils, but would not produce crepitus. A perforated septum would also not produce crepitus.

A client has an S3 heart sound. The nurse knows: Hint: Anatomy and Physiology Review, Heart Sounds An S3 heart sound is indicative of heart disease in almost all individuals. An S3 heart sound is a systolic sound. A physiologic S3 is common in children and young adults. The presence of an S3 heart sound is an expected and normal finding in the elderly client.

A physiologic S3 is common in children and young adults. An S3 heart sound, also called a ventricular gallop, occurs early in diastole when blood is flowing from the atria into the ventricles and causes vibrations. S3 is a physiologic heart sound in children, young adults, and pregnant females in their third trimester. The presence of an S3 heart sound may be associated with pathologic conditions such as myocardial infarction (MI) or heart failure; however, it is not indicative of heart disease in the majority of individuals. The S3 heart sound is an early diastolic sound. The presence of an S3 sound is not an expected sound in an elderly adult and may signify the presence of heart failure or a previous MI.

Walking contracts the calf muscles and forces blood away from the heart.

false

26. During the precordial assessment on an 8-month pregnant patient, the nurse palpates the apical impulse at the fourth left intercostal space lateral to the midclavicular line. This would indicate 1. right ventricular hypertrophy. 2. increased volume and size of the heart as a result of pregnancy. 3. displacement of the heart from elevation of the diaphragm. 4. increased blood flow through the internal mammary artery.

ANS: 3) displacement of the heart from elevation of the diaphragm. Palpation of the apical impulse is higher and lateral compared with the normal position because the enlarging uterus elevates the diaphragm and displaces the heart up and to the left and rotates it on its long axis.

During a cardiovascular assessment, the nurse knows that an S4 heart sound is: 1. heard at the onset of atrial diastole. 2. usually a normal finding in the elderly. 3. heard at the end of ventricular diastole. 4. heard best over the second left intercostal space with the individual sitting upright.

ANS: 3) heard at the end of ventricular diastole. An S4 heart sound is heard at the end of diastole when the atria contract (atrial systole) and when the ventricle is resistant to filling. The S4 occurs just before the S1.

In performing auscultation of heart sounds, which sequence would the nurse use 1. Aortic area—pulmonic area—Erb's point—tricuspid area- mitral area 2. Pulmonic area—aortic area—Erb's point—tricuspid area—mitral area 3. Aortic area—tricuspid area—Erb's point—mitral area—pulmonic area 4. Pulmonic area—Erb's point—tricuspid area—pulmonic area—mitral area

ANS: 1) Aortic area—pulmonic area—Erb's point—tricuspid area—mitral area Sounds produced by the valves may be heard all over the precordium. Therefore, learn to inch your stethoscope in a Z pattern, from the base of the heart across and down, and then over to the apex. Or start at the apex and work your way up.

When assessing a patient's lungs, the nurse recalls that the left lung: A) consists of two lobes. B) is divided by the horizontal fissure. C) consists primarily of an upper lobe on the posterior chest. D) is shorter than the right lung because of the underlying stomach.

ANS: A The left lung has two lobes, and the right lung has three lobes. The right lung is shorter than the left lung because of the underlying liver. The left lung is narrower than the right lung because the heart bulges to the left. The posterior chest is almost all lower lobe.

The mother of a 10-month-old tells the nurse that she has noticed that her son becomes blue when he is crying and that the frequency of this is increasing. He is also not crawling yet. During the examination the nurse palpates a thrill at the left lower sternal border and auscultates a loud systolic murmur in the same area. What would be the most likely cause of these findings? 1. Tetralogy of Fallot 2. Atrial septal defect 3. Patent ductus arteriosus 4. Ventricular septal defect

ANS: 1) Tetralogy of fallot Tetralogy of Fallot subjective findings include (1) severe cyanosis, not in the first months of life but developing as the infant grows, and right ventricle outflow (i.e., pulmonic) stenosis gets worse; (2) cyanosis with crying and exertion at first, then at rest; (3) slowed development. Objective findings include (1) thrill palpable at left lower sternal border; (2) S1 normal, S2 has A2 loud and P2 diminished or absent; (3) murmur is systolic, loud, crescendo-decrescendo.

A 25-year-old woman in her fifth month of pregnancy has a blood pressure of 100/70 mm Hg. In reviewing her previous exam, the nurse notes that her blood pressure in her second month was 124/80 mm Hg. In evaluating this change, what does the nurse know to be true? 1. This is the result of peripheral vasodilatation and is an expected change. 2. Because of increased cardiac output, the blood pressure should be higher this time. 3. This is not an expected finding because it would mean a decreased cardiac output. 4. This would mean a decrease in circulating blood volume, which is dangerous for the fetus.

ANS: 1) This is the result of peripheral vasodilatation and is an expected change. Despite the increased cardiac output, arterial blood pressure decreases in pregnancy because of peripheral vasodilatation. The blood pressure drops to its lowest point during the second trimester and then rises after that.

During a cardiovascular assessment, the nurse knows that a "thrill" is: 1. a vibration that is palpable. 2. palpated in the right epigastric area. 3. associated with ventricular hypertrophy. 4. a murmur auscultated at the third intercostal space.

ANS: 1) a vibration that palpable A thrill is a palpable vibration. It signifies turbulent blood flow and accompanies loud murmurs. The absence of a thrill does not rule out the presence of a murmur.

Which racial group has the highest prevalence of heart disease and stroke in the United States? 1. Blacks 2. Whites 3. American Indians 4. Mexican-Americans

ANS: 1) blacks According to the American Heart Association, the prevalence of heart disease and stroke is higher among black adults than in other racial groups.

The nurse is assessing the apical pulse of a 3-month-old infant and finds the rate to be 135 beats per minute. The nurse interprets this result as: 1. normal for this age. 2. lower than expected. 3. higher than expected, probably as a result of crying. 4. higher than expected, reflecting persistent tachycardia.

ANS: 1) normal for his age The heart rate may range from 100 to 180 beats per minute immediately after birth and then stabilize to an average of 120 to 140 beats per minute. Infants normally have wide fluctuations with activity, from 170 beats per minute or more with crying or being active to 70 to 90 beats per minute with sleeping. Persistent tachycardia is greater than 200 beats per minute in newborns or greater than 150 beats per minute in infants.

The sac that surrounds and protects the heart is called the: 1. pericardium. 2. myocardium. 3. endocardium. 4. pleural space.

ANS: 1) pericardium The pericardium is a tough fibrous double-walled sac that surrounds and protects the heart. It has two layers that contain a few milliliters of serous pericardial fluid.

In percussing the left cardiac border, the nurse would expect to hear dullness at the 1. third left intercostal space midclavicular line and fifth left intercostal space left sternal border. 2. fourth left intercostal space medial to midclavicular line and second left intercostal space midclavicular line. 3. fifth left intercostal space midclavicular line and second left intercostal space sternal border. 4. fifth left intercostal space sternal border and second right intercostal space midclavicular line.

ANS: 1) third left intercostal space midclavicular line and fifth left intercostal space left sternal border. Normally, the left border of cardiac dullness is at the midclavicular line in the fifth interspace and slopes in toward the sternum as you progress upward so that by the second interspace the border of dullness coincides with the left sternal border.

How should the nurse document mild, slight pitting edema present at the ankles of a pregnant patient?

ANS: 1+/0-4+ If pitting edema is present, then the nurse should grade it on a scale of 1+ (mild) to 4+ (severe). Brawny edema appears as nonpitting edema and feels hard to the touch.

In assessing a 70-year-old man, the nurse finds the following: BP 140/100 mm Hg; HR 104 and slightly irregular; split S2. Which of these findings can be explained by expected hemodynamic changes related to age? 1. Increase in resting heart rate 2. Increase in systolic blood pressure 3. Decrease in diastolic blood pressure 4. Increase in diastolic blood pressure

ANS: 2) Increase in systolic blood pressure From ages 20 to 80 years, systolic blood pressure tends to increase within the normal range by 25% to 30%. No significant change in diastolic pressure occurs with age. No change in resting heart rate occurs with aging. Cardiac output at rest is not changed with aging.

The direction of blood flow through the heart is best described by which of the following? 1. Vena cava—right atrium—right ventricle—lungs—pulmonary artery—left atrium —left ventricle 2. Right atrium—right ventricle—pulmonary artery—lungs—pulmonary vein—left atrium—left ventricle 3. Aorta—right atrium—right ventricle—lungs—pulmonary vein—left atrium—left ventricle—vena cava 4. Right atrium—right ventricle—pulmonary vein—lungs—pulmonary artery—left atrium—left ventricle

ANS: 2) Right atrium—right ventricle—pulmonary artery—lungs—pulmonary vein—left atrium—left ventricle Returning blood from the body empties into the right atrium and flows into the right ventricle and then goes to the lungs through the pulmonary artery. The lungs oxygenate the blood and it is then returned to the left atrium by the pulmonary vein. It goes from there to the left ventricle and then out to the body through the aorta.

The primary muscles of respiration include the: A) diaphragm and intercostals. B) sternomastoids and scaleni. C) trapezius and rectus abdominis. D) external obliques and pectoralis major.

ANS: A The major muscle of respiration is the diaphragm. The intercostal muscles lift the sternum and elevate the ribs during inspiration, increasing the anteroposterior diameter. Expiration is primarily passive. Forced inspiration involves the use of other muscles, such as the accessory neck muscles (sternomastoids, scalene, trapezii). Forced expiration involves the abdominal muscles.

The nurse is assessing a patient with possible cardiomyopathy and assesses the hepatojugular reflux. If heart failure is present, the nurse would see which finding while pushing on the right upper quadrant of the patient's abdomen, just below the rib cage? 1. The jugular veins will rise for a few seconds and then recede back to the previous level if the heart is working properly. 2. The jugular veins will remain elevated as long as pressure on the abdomen is maintained. 3. An impulse will be visible at the fourth or fifth intercostal space, at or inside the midclavicular line. 4. The jugular veins will not be detected during this maneuver.

ANS: 2) The jugular veins will remain elevated as long as pressure on the abdomen is maintained. When performing hepatojugular reflux, the jugular veins will rise for a few seconds and then recede back to the previous level if the heart is able to pump the additional volume created by the pushing; however, with heart failure, the jugular veins remain elevated as long as pressure on the abdomen is maintained.

While auscultating heart sounds on a 7-year-old for a "routine physical," the nurse hears the following: an S3, a soft murmur at left midsternal border, and a venous hum when the child is standing. Which of the following would be true regarding the findings? 1. S3 is indicative of heart disease in children. 2. These can all be normal findings in a child. 3. These are indicative of congenital problems. 4. The venous hum most likely indicates an aneurysm.

ANS: 2) These can all be normal findings in a child. Physiologic S3 is common in children. A venous hum, caused by turbulence of blood flow in the jugular venous system, is common in healthy children and has no pathologic significance. Heart murmurs that are innocent (or functional) in origin are very common through childhood.

The vital signs of a 70-year-old patient with a history of hypertension are BP 180/100 and HR 90. The nurse hears an extra heart sound at the apex immediately before S1. The sound is heard only with the bell while patient is in left lateral position. With these findings and the patient's history, the nurse knows that this extra heart sound is most likely: 1. split S1. 2. atrial gallop. 3. diastolic murmur. 4. summation sound.

ANS: 2) atrial gallop A pathologic S4 is termed an atrial gallop or an S4 gallop. It occurs with decreased compliance of the ventricle and with systolic overload (afterload), including outflow obstruction to the ventricle (aortic stenosis) and systemic hypertension. A left-sided S4 occurs with these conditions. It is heard best at the apex with the patient in the left lateral position.

In assessing for an S4 with a stethoscope, the nurse would listen with the: 1. bell at the base with the patient leaning forward. 2. bell at the apex with the patient in the left lateral position. 3. diaphragm in the aortic area with the patient sitting. 4. diaphragm in the pulmonic area with the patient supine.

ANS: 2) bell at the apex with the patient in the left lateral position. The S4 is a ventricular filling sound. It occurs when atria contract late in diastole. It is heard immediately before S1. This is a very soft sound, of very low pitch. You need a good bell, and you must listen for it. It is heard best at the apex, with the person in the left lateral position.

During an assessment of a 68-year-old man with a recent onset of right- sided weakness, the nurse hears a blowing, swishing sound with the bell of the stethoscope over the left carotid artery. This finding would indicate: 1. a valvular disorder. 2. blood flow turbulence. 3. fluid volume overload. 4. ventricular hypertrophy.

ANS: 2) blood flow turbulence. A bruit is a blowing, swishing sound indicating blood flow turbulence; normally none is present

A heart sound heard during the interval between the second heart sound (S2) and the next first sound (S1) is a(n): 1. systolic sound. 2. diastolic sound. 3. atrial filling sound. 4. ventricular contraction sound.

ANS: 2) diastolic sound. S2 signifies the onset of diastole. Any sound heard between S2 and the next first heart sound (S1) is a diastolic sound. A sound heard between S1 and S2 is a systolic sound.

The nurse knows that normal splitting of the second heart sound is associated with: 1. expiration. 2. inspiration. 3. exercise state. 4. low resting heart rate.

ANS: 2) inspiration Normal or physiologic splitting of the second heart sound is associated with inspira- tion because of the increased blood return to the right side of the heart, delaying closure of the pulmonic valve.

In assessing the carotid arteries of an older patient with cardiovascular disease, the nurse would: 1. palpate the artery in the upper one third of the neck. 2. listen with the bell of the stethoscope to assess for bruits. 3. palpate both arteries simultaneously to compare amplitude. 4. instruct patient to take slow deep breaths during auscultation.

ANS: 2) listen with the bell of the stethoscope to assess for bruits. If cardiovascular disease is suspected, auscultate each carotid artery for the presence of a bruit. Avoid compressing the artery because this could create an artificial bruit and it could compromise circulation if the carotid artery is already narrowed by atherosclerosis.

The component of the conduction system referred to as the pacemaker of the heart is the: 1. atrioventricular (AV) node. 2. sinoatrial (SA) node. 3. bundle of His. 4. bundle branches.

ANS: 2) sinoatrial (SA) node. Specialized cells in the SA node near the superior vena cava initiate an electrical impulse. (Because the SA node has an intrinsic rhythm, it is the "pacemaker.")

A 45-year-old man is in the clinic for "a routine physical." During the history the patient states he's been having difficulty sleeping. "I'll be sleeping great and then I wake up and feel like I can't get my breath." The nurse's best response to this would be: 1. "When was your last electrocardiogram?" 2. "It's probably because it's been so hot at night." 3. "Do you have any history of problems with your heart?" 4. "Have you had a recent sinus infection or upper respiratory infection?"

ANS: 3) "Do you have any history of problems with your heart?" Paroxysmal nocturnal dyspnea occurs with heart failure. Lying down increases volume of intrathoracic blood, and the weakened heart cannot accommodate the increased load. Classically, the person awakens after 2 hours of sleep, arises, and flings open a window with the perception of needing fresh air.

Which of the following statements is true regarding the apical impulse? 1. It is palpable in all adults. 2. It occurs with the onset of diastole. 3. Its location may be indicative of heart size. 4. It should normally be palpable in the anterior axillary line.

ANS: 3) Its location may be indicative of heart size. The apical impulse is palpable in about 50% of adults. It is located in the fifth left intercostal space in the midclavicular line. Horizontal or downward displacement of the apical impulse may indicate an enlargement of the left ventricle.

When the nurse is auscultating the carotid artery for bruits, which of the following reflects correct technique? 1. While listening with the bell of the stethoscope, have the patient take a deep breath and hold it. 2. While auscultating one side with the bell of the stethoscope, palpate the carotid artery on the other side to check pulsations. 3. Lightly apply the bell of the stethoscope over the carotid artery; have the patient take a breath, exhale, and hold it briefly while the nurse listens. 4. Firmly place the bell of the stethoscope over the carotid artery, have the patient take a breath, exhale, and hold it briefly while the nurse listens.

ANS: 3) Lightly apply the bell of the stethoscope over the carotid artery; have the patient take a breath, exhale, and hold it briefly while the nurse listens. Lightly apply the bell of the stethoscope over the carotid artery at three levels; have the patient take a breath, exhale, and hold it briefly while you listen. Holding the breath on inhalation will also tense the levator scapulae muscles, which makes it hard to hear the carotids. Examine only one carotid artery at a time to avoid compromising arterial blood flow to the brain. Avoid pressure over the carotid sinus, which may lead to decreased heart rate, decreased blood pressure, and cerebral ischemia with syncope.

Which of the following would the nurse expect to find during a cardiac assessment on a 4-year-old child? 1. S3 when sitting up 2. Persistent tachycardia above 150 3. Murmur at second left intercostal space when supine 4. Palpable apical impulse in fifth left intercostal space lateral to midclavicular line

ANS: 3) Murmur at second left intercostal space when supine Some murmurs are common in healthy children or adolescents and are termed innocent or functional. The contractile force of the heart is greater in children. This increases blood flow velocity. The increased velocity plus a smaller chest measure- ment makes an audible murmur. The innocent murmur is heard at the second or third left intercostal space and disappears with sitting, and the young person has no associated signs of cardiac dysfunction.

While counting the apical pulse on a 16-year-old patient, the nurse notes an irregular rhythm. His rate speeds up on inspiration and slows on expiration. What would be the nurse's response? 1. Talk with the patient about his intake of caffeine. 2. Do an electrocardiogram after the exam. 3. No further response is needed because this is normal. 4. Refer the patient to a cardiologist for further testing.

ANS: 3) No further response is needed because this is normal. The rhythm should be regular, although sinus arrhythmia occurs normally in young adults and children. With sinus arrhythmia, the rhythm varies with the person's breathing, increasing at the peak of inspiration, and slowing with expiration.

When listening to heart sounds, the nurse knows that which of the following statements concerning S1 is true? 1. S1 is louder than S2 at the base. 2. S1 indicates the beginning of diastole. 3. S1 coincides with the carotid artery pulse. 4. S1 is caused by closure of the semilunar valves.

ANS: 3) S1 coincides with the carotid artery pulse. S1 is the start of systole, and is louder than S2 at the apex; S2 is louder than S1 at the base. S1 coincides with carotid artery pulse. Feel the carotid gently as you auscultate at the apex; the sound you hear as you feel each pulse is S1.

The electrical stimulus of the cardiac cycle follows which sequence? 1. AV node—SA node—bundle of His 2. Bundle of His—AV node—SA node 3. SA node—AV node—bundle of His—bundle branches 4. AV node—SA node—bundle of His—bundle branches

ANS: 3) SA node—AV node—bundle of His—bundle branches Specialized cells in the SA node near the superior vena cava initiate an electrical impulse. The current flows in an orderly sequence, first across the atria to the AV node low in the atrial septum. There it is delayed slightly so that the atria have time to contract before the ventricles are stimulated. Then the impulse travels to the bundle of His, the right and left bundle branches, and then through the ventricles.

In assessing a patient's major risk factors for heart disease, which would the nurse want to include when taking a history? 1. Family history, hypertension, stress, age 2. Personality type, high cholesterol, diabetes, smoking 3. Smoking, hypertension, obesity, diabetes, high cholesterol 4. Alcohol consumption, obesity, diabetes, stress, high cholesterol

ANS: 3) Smoking, hypertension, obesity, diabetes, high cholesterol For major risk factors for coronary artery disease, collect data regarding elevated serum cholesterol, elevated blood pressure, blood glucose levels above 130 mg/dl or known diabetes mellitus, obesity, cigarette smoking, low activity level.

Which of the following describes the closure of the valves in a normal cardiac cycle? 1. The aortic valve closes slightly before the tricuspid valve. 2. The pulmonic valve closes slightly before the aortic valve. 3. The tricuspid valve closes slightly later than the mitral valve. 4. Both the tricuspid and pulmonic valves close at the same time.

ANS: 3) The tricuspid valve closes slightly later than the mitral valve. Events occur just slightly later in the right side of the heart because of the route of myocardial depolarization. As a result, two distinct components to each of the heart sounds exist, and sometimes you can hear them separately. In the first heart sound, the mitral component (M1) closes just before the tricuspid component (T1).

When assessing a newborn infant who is just 5 minutes old, the nurse knows that which of the following would be true? 1. The left ventricle is larger and weighs more than the right. 2. The circulation of a newborn is identical to that of an adult. 3. There is an opening in the atrial septum where blood can flow into the left side of the heart. 4. The foramen ovale closes just minutes before birth and the ductus arteriosus closes immediately after.

ANS: 3) There is an opening in the atrial septum where blood can flow into the left side of the heart. First, about two thirds of the blood is shunted through an opening in the atrial septum, the foramen ovale into the left side of the heart, where it is pumped out through the aorta. The foramen ovale closes within the first hour because the pressure in the right side of the heart is now lower than in the left side.

When listening to heart sounds, the nurse knows that the valve closures that can be heard best at the base of the heart are: 1. mitral, tricuspid. 2. tricuspid, aortic. 3. aortic, pulmonic. 4. mitral, pulmonic.

ANS: 3) aortic, pulmonic. The second heart sound (S2) occurs with closure of the semilunar valves and signals the end of systole. Although it is heard over all the precordium, S2 is loudest at the base.

The nurse is performing a cardiac assessment on a 65-year-old patient 3 days after her myocardial infarction. Heart sounds are normal when she is supine, but with the patient sitting and leaning forward, the nurse hears a high-pitched, scratchy sound at the apex with the diaphragm. It disappears on inspiration. The nurse suspects: 1. increased cardiac output. 2. another myocardial infarction. 3. inflammation of the precordium. 4. ventricular hypertrophy resulting from muscle damage.

ANS: 3) inflammation of the precordium. Inflammation of the precordium gives rise to a friction rub. The sound is high pitched and scratchy, like sandpaper being rubbed: It is best heard with the diaphragm, with the person sitting up and leaning forward, and with the breath held in expiration. A friction rub can be heard any place on the precordium but usually is best heard at the apex and left lower sternal border, places where the pericardium comes in close contact with the chest wall.

During an assessment of a healthy adult, where would the nurse expect to palpate the apical impulse? 1. Third left intercostal space at the midclavicular line 2. Fourth left intercostal space at the sternal border 3. Fourth left intercostal space at the anterior axillary line 4. Fifth left intercostal space at the midclavicular line

ANS: 4) Fifth left intercostal space at the midclavicular line Location—the apical impulse should occupy only one interspace, the fourth or fifth, and be at or medial to the midclavicular line.

The mother of a 3-month-old states that her daughter has not been gaining weight. With further questioning, the nurse finds that the infant falls asleep after nursing and wakes up after a short amount of time, hungry again. What other information would the nurse want to have? 1. The position that baby sleeps in 2. Sibling history of eating disorders 3. Amount of background noise when eating 4. Presence of dyspnea or diaphoresis when sucking

ANS: 4) Presence of dyspnea or diaphoresis when sucking To screen for heart disease in an infant, focus on feeding. Note fatigue during feeding. Infant with heart failure takes fewer ounces each feeding, becomes dyspneic with sucking, may be diaphoretic and then falls into exhausted sleep and awakens after a short time hungry again.

Which of the following best describes what is meant by atrial kick? 1. The atria contract during systole and attempt to push against closed valves. 2. The contraction of the atria at the beginning of diastole can be felt as a palpitation. 3. This is the pressure exerted against the atria as the ventricles contract during systole. 4. The atria contract toward the end of diastole and push the remaining blood into the ventricles.

ANS: 4) The atria contract toward the end of diastole and push the remaining blood into the ventricles. Toward the end of diastole, the atria contract and push the last amount of blood (about 25% of stroke volume) into the ventricles. This active filling phase is called presystole, or atrial systole, or sometimes the "atrial kick."

During the cardiac auscultation the nurse hears a sound occurring immediately after S2 at the second left intercostal space. To further assess this sound, what would the nurse do? 1. Have patient turn to the left side and listen with the bell. 2. Ask patient to hold his breath while the nurse listens again. 3. No further assessment is needed because the nurse knows it is an S3. 4. Watch patient's respirations while listening for effect on the sound.

ANS: 4) Watch patient's respirations while listening for effect on the sound. A split S2 is a normal phenomenon that occurs toward the end of inspiration in some people. A split S2 is heard only in the pulmonic valve area, the second left interspace. When you first hear the split S2, do not be tempted to ask the person to hold his or her breath so that you can concentrate on the sounds. Breath holding will only equalize ejection times in the right and left sides of the heart and cause the split to go away. Instead, concentrate on the split as you watch the person's chest rise up and down with breathing.

During inspection of the precordium of an adult patient, the nurse notices the chest moving in a forceful manner along the fourth-fifth left intercostal space at the midclavicular line. This finding most likely suggests: 1. a normal heart. 2. a systolic murmur. 3. enlargement of the left ventricle. 4. enlargement of the right ventricle.

ANS: 4) enlargement of the right ventricle. A heave or lift is a sustained forceful thrusting of the ventricle during systole. It occurs with ventricular hypertrophy as a result of increased workload. A right ventricular heave is seen at the sternal border; a left ventricular heave is seen at the apex.

During a cardiac assessment on an adult patient in the hospital for "chest pain," the nurse finds the following: jugular vein pulsations 4 cm above sternal angle when he is elevated at 45 degrees, BP 98/60, HR 130; ankle edema; difficulty in breathing when supine; and an S3 on auscultation. Which of the following best explains the cause of these findings? 1. Fluid overload 2. Atrial septal defect 3. Myocardial infarction 4. Heart failure

ANS: 4) heart failure Heart failure causes decreased cardiac output when the heart fails as a pump and the circulation becomes backed up and congested. Signs and symptoms: dyspnea, orthopnea, paroxysmal nocturnal dyspnea, decreased blood pressure, dependent, pitting edema; anxiety; confusion; jugular vein distention; fatigue. The S3 may be the earliest sign of heart failure.

The findings from an assessment of a 70-year-old patient with swelling in his ankles include jugular venous pulsations 5 cm above the sternal angle when the head of his bed is elevated 45 degrees. The nurse knows that this finding indicates: 1. decreased fluid volume. 2. increased cardiac output. 3. narrowing of jugular veins. 4. increased pressure in the right side of his heart.

ANS: 4) increased pressure in the right side of his heart. Because no cardiac valve exists to separate the superior vena cava from the right atrium, the jugular veins give information about activity on the right side of the heart. They reflect filling pressures and volume changes. Normal jugular venous pulsation is 2 cm or less above the sternal angle. Elevated pressure is more than 3 cm above the sternal angle at 45 degrees and occurs with right-sided heart failure.

Which of these statements is true regarding the vertebra prominens? The vertebra prominens is: A) the spinous process of C7. B) usually not palpable in most individuals. C) opposite the interior border of the scapula. D) located next to the manubrium of the sternum.

ANS: A The spinous process of C7 is the vertebra prominens. It is the most prominent bony spur protruding at the base of the neck. Counting ribs and intercostal spaces on the posterior thorax is difficult because of the muscles and soft tissue. The vertebra prominens is easier to identify and is used as a starting point in counting thoracic processes and identifying landmarks on the posterior chest.

A 30-year-old woman with a history of mitral valve problems states that she has been "very tired." She has started waking up at night and feels like her "heart is pounding." During the assessment, the nurse palpates a thrill and lift at the fifth left intercostal space midclavicular line. In the same area the nurse also auscultates a blowing, swishing sound right after S1. These findings would be most consistent with: 1. heart failure. 2. aortic stenosis. 3. pulmonary edema. 4. mitral regurgitation.

ANS: 4) mitral regurgitation Mitral regurgitation subjective findings include fatigue, palpitation, and orthopnea. Objective findings are (1) a thrill in systole at apex, (2) lift at apex, (3) apical impulse displaced down and to the left, (4) S1 diminished, S2 accentuated, S3 at apex often present, and (5) murmur: pansystolic, often loud, blowing, best heard at apex, radiating well to the left axilla.

During an assessment, the nurse notes that the apical impulse is displaced laterally, and it is palpable over a wide area. This indicates: 1. systemic hypertension. 2. pulmonic hypertension. 3. pressure overload, as in aortic stenosis. 4. volume overload, as in mitral regurgitation.

ANS: 4) volume overload, as in mitral regurgitation. Cardiac enlargement displaces the apical impulse laterally and over a wider area when left ventricular hypertrophy and dilatation are present. This is volume overload, as in mitral regurgitation, aortic regurgitation, or left-to-right shunts.

The nurse is percussing over the lungs of a patient with pneumonia. The nurse knows that percussion over an area of atelectasis in the lungs would reveal: A) dullness. B) tympany. C) resonance. D) hyperresonance.

ANS: A A dull percussion note signals an abnormal density in the lungs, as with pneumonia, pleural effusion, atelectasis, or tumor.

An adult patient with a history of allergies comes to the clinic complaining of wheezing and difficulty in breathing when working in his yard. The assessment findings include tachypnea, use of accessory neck muscles, prolonged expiration, intercostal retractions, decreased breath sounds, and expiratory wheezes. The nurse interprets that these assessment findings are consistent with: A) asthma. B) atelectasis. C) lobar pneumonia. D) heart failure.

ANS: A Asthma is allergic hypersensitivity to certain inhaled particles that produces inflammation and a reaction of bronchospasm, which increases airway resistance, especially during expiration. Increased respiratory rate, use of accessory muscles, retraction of intercostal muscles, prolonged expiration, decreased breath sounds, and expiratory wheezing are all characteristic of asthma. See Table 18-8 for descriptions of the other conditions.

During auscultation of the lungs, the nurse expects decreased breath sounds to be heard in which situation? A) When the bronchial tree is obstructed B) When adventitious sounds are present C) In conjunction with whispered pectoriloquy D) In conditions of consolidation, such as pneumonia

ANS: A Decreased or absent breath sounds occur when the bronchial tree is obstructed, as in emphysema, and when sound transmission is obstructed, as in pleurisy, pneumothorax, or pleural effusion.

During auscultation of breath sounds, the nurse should use the stethoscope correctly, in which of the following ways? A) Listen to at least one full respiration in each location. B) Listen as the patient inhales and then go to the next site during exhalation. C) Have the patient breathe in and out rapidly while the nurse listens to the breath sounds. D) If the patient is modest, listen to sounds over his or her clothing or hospital gown.

ANS: A During auscultation of breath sounds with a stethoscope, it is important to listen to one full respiration in each location. During the examination, the nurse should monitor the breathing and offer times for the person to breathe normally to prevent possible dizziness

A 70-year-old patient is being seen in the clinic for severe exacerbation of his heart failure. Which of these findings is the nurse most likely to observe in this situation? A) Shortness of breath, orthopnea, paroxysmal nocturnal dyspnea, ankle edema B) Rasping cough, thick mucoid sputum, wheezing, bronchitis C) Productive cough, dyspnea, weight loss, anorexia, tuberculosis D) Fever, dry nonproductive cough, diminished breath sounds

ANS: A Heart failure often presents with increased respiratory rate, shortness of breath on exertion, orthopnea, paroxysmal nocturnal dyspnea, nocturia, ankle edema, and pallor in light-skinned individuals. A patient with rasping cough, thick mucoid sputum, and wheezing may have bronchitis. Productive cough, dyspnea, weight loss, and dyspnea are seen with tuberculosis; fever, dry nonproductive cough, and diminished breath sounds may indicate Pneumocystis jiroveci (P. carinii) pneumonia. See Table 18-8.

When assessing tactile fremitus, the nurse recalls that it is normal to feel tactile fremitus most intensely over which location? A) Between the scapulae B) Third intercostal space, MCL C) Fifth intercostal space, MAL D) Over the lower lobes, posterior side

ANS: A Normally, fremitus is most prominent between the scapulae and around the sternum. These are sites where the major bronchi are closest to the chest wall. Fremitus normally decreases as one progress down the chest because more tissue impedes sound transmission.

The nurse is auscultating the lungs of a patient who had been sleeping and notices short, popping, crackling sounds that stop after a few breaths. The nurse recognizes that these breath sounds are: A) atelectatic crackles, and that they are not pathologic. B) fine crackles, and that they may be a sign of pneumonia. C) vesicular breath sounds. D) fine wheezes.

ANS: A One type of adventitious sound, atelectatic crackles, is not pathologic. They are short, popping, crackling sounds that sound like fine crackles but do not last beyond a few breaths. When sections of alveoli are not fully aerated (as in people who are asleep or in the elderly), they deflate slightly and accumulate secretions. Crackles are heard when these sections are expanded by a few deep breaths. Atelectatic crackles are heard only in the periphery, usually in dependent portions of the lungs, and disappear after the first few breaths or after a cough.

The nurse is observing the auscultation technique of another nurse. The correct method to use when progressing from one auscultatory site on the thorax to another is ____ comparison. A) side-to-side B) top-to-bottom C) posterior-to-anterior D) interspace-by-interspace

ANS: A Side-to-side comparison is most important when auscultating the chest. The nurse should listen to at least one full respiration in each location. The other techniques are incorrect.

The nurse is assessing the pulses of a patient who has been admitted for untreated hyperthyroidism. The nurse should expect to find a(n) _____ pulse.

ANS: bounding A full, bounding pulse occurs with hyperkinetic states (such as exercise, anxiety, fever), anemia, and hyperthyroidism. Absent pulse occurs with occlusion. Weak, thready pulses occur with shock and peripheral artery disease.

The nurse is listening to the breath sounds of a patient with severe asthma. Air passing through narrowed bronchioles would produce which of these adventitious sounds? A) Wheezes B) Bronchial sounds C) Bronchophony D) Whispered pectoriloquy

ANS: A Wheezes are caused by air squeezed or compressed through passageways narrowed almost to closure by collapsing, swelling, secretions, or tumors, such as with acute asthma or chronic emphysema.

A patient has been diagnosed with venous stasis. Which of these findings would the nurse most likely observe?

ANS: A brownish discoloration to the skin of the lower leg A brown discoloration occurs with chronic venous stasis as a result of hemosiderin deposits (a by-product of red blood cell degradation). Pallor, cyanosis, atrophic skin, and unilateral coolness are all signs associated with arterial problems.

The nurse is preparing to perform a manual compression test on a patient. Which of these statements is true about this procedure?

ANS: A palpable wave transmission occurs when the valves are incompetent. With the manual compression test, a palpable wave transmission occurs when the valves are incompetent. Competent veins will prevent a wave transmission and the nurse's distal (lower) fingers will feel no change. The test is performed while the patient is standing.

The nurse is assessing voice sounds during a respiratory assessment. Which of these findings indicates a normal assessment? Select all that apply. A) Voice sounds are faint, muffled, and almost inaudible when the patient whispers "one, two, three" in a very soft voice. B) As the patient says "ninety-nine" repeatedly, the examiner hears the words "ninety-nine" clearly. C) When the patient speaks in a normal voice, the examiner can hear a sound but cannot distinguish exactly what is being said. D) As the patient says a long "ee-ee-ee" sound, the examiner also hears a long "ee-ee-ee" sound. E) As the patient says a long "ee-ee-ee" sound, the examiner hears a long "aaaaaa" sound.

ANS: A, C, D As a patient says "ninety-nine" repeatedly, normally, the examiner hears sound but cannot distinguish what is being said. If a clear "ninety-nine" is auscultated, then it could indicate increased lung density, which enhances transmission of voice sounds. This is a measure of bronchophony. When a patient says a long "ee-ee-ee" sound, normally the examiner also hears a long "ee-ee-ee" sound through auscultation. This is a measure of egophony. If the examiner hears a long "aaaaaa" sound instead, this could indicate areas of consolidation or compression. With whispered pectoriloquy, as when a patient whispers a phrase such as "one-two-three," the normal response when auscultating voice sounds is to hear sounds that are faint, muffled, and almost inaudible. If the examiners hears the whispered voice clearly, as if the patient is speaking through the stethoscope, then consolidation of the lung fields may exist.

The nurse is preparing to assess the ankle-brachial index (ABI) of a patient. Which statement about the ABI is true?

ANS: An ABI of 0.90 to 0.70 indicates the presence of peripheral vascular disease and mild claudication. Use of the Doppler stethoscope is a noninvasive way to determine the extent of peripheral vascular disease. The normal ankle pressure is slightly greater than or equal to the brachial pressure. An ABI of 0.90 to 0.70 indicates the presence of peripheral vascular disease and mild claudication. The ABI is less reliable in patients with diabetes mellitus because of claudication, which makes the arteries noncompressible and may give a falsely high ankle pressure.

The nurse is attempting to assess the femoral pulse in an obese patient. Which of these actions would be most appropriate?

ANS: Ask the patient to bend his or her knees to the side in a froglike position. To help expose the femoral area, particularly in obese people, the nurse should ask the person to bend his or her knees to the side in a froglike position.

When assessing a patient the nurse notes that the left femoral pulse as diminished, 1+/4+. What should the nurse do next?

ANS: Auscultate the site for a bruit. If a pulse is weak or diminished at the femoral site, the nurse should auscultate for a bruit. Presence of a bruit, or turbulent blood flow, indicates partial occlusion. The other responses are not correct.

A patient comes to the clinic complaining of a cough that is worse at night but not as bad during the day. The nurse recognizes that this may indicate: A) pneumonia. B) postnasal drip or sinusitis. C) exposure to irritants at work. D) chronic bronchial irritation from smoking.

ANS: B A cough that occurs mainly at night may indicate postnasal drip or sinusitis. Exposure to irritants at work causes an afternoon or evening cough. Smokers experience early morning coughing. Coughing associated with acute illnesses such as pneumonia is continuous throughout the day.

A patient with pleuritis has been admitted to the hospital and complains of pain with breathing. What other key assessment finding would the nurse expect to find upon auscultation? A) Stridor B) Friction rub C) Crackles D) Wheezing

ANS: B A patient with pleuritis will exhibit a pleural friction rub upon auscultation. This is the sound made when pleurae become inflamed and rub together during respiration. The sound is superficial, coarse, and low-pitched, as if two pieces of leather are being rubbed together. Stridor is associated with croup, acute epiglottitis in children, and foreign body inhalation. Crackles are associated with several diseases, such as pneumonia, heart failure, chronic bronchitis, and others (see Table 18-6). Wheezes are associated with diffuse airway obstruction caused by acute asthma or chronic emphysema.

During palpation of the anterior chest wall, the nurse notices a coarse, crackling sensation over the skin surface. On the basis of these findings, the nurse suspects: A) tactile fremitus. B) crepitus. C) friction rub. D) adventitious sounds.

ANS: B Crepitus is a coarse, crackling sensation palpable over the skin surface. It occurs in subcutaneous emphysema when air escapes from the lung and enters the subcutaneous tissue, as after open thoracic injury or surgery.

Which statement about the apices of the lungs is true? The apices of the lungs: A) are at the level of the second rib anteriorly. B) extend 3 to 4 cm above the inner third of the clavicles. C) are located at the sixth rib anteriorly and the eighth rib laterally. D) rest on the diaphragm at the fifth intercostal space in the midclavicular line.

ANS: B The apex of the lung on the anterior chest is 3 to 4 cm above the inner third of the clavicles. On the posterior chest, the apices are at the level of C7.

The nurse is reviewing the blood supply to the arm. The major artery supplying the arm is the _____ artery.

ANS: brachial The major artery supplying the arm is the brachial artery. The brachial artery bifurcates into the ulnar and radial arteries immediately below the elbow. In the hand, the ulnar and radial arteries form two arches known as the superficial and deep palmar arches.

The nurse knows that a normal finding when assessing the respiratory system of an elderly adult is: A) increased thoracic expansion. B) decreased mobility of the thorax. C) a decreased anteroposterior diameter. D) bronchovesicular breath sounds throughout the lungs.

ANS: B The costal cartilages become calcified with aging, resulting in a less mobile thorax. Chest expansion may be somewhat decreased, and the chest cage commonly shows an increased anteroposterior diameter.

A mother brings her 3-month-old infant to the clinic for evaluation of a cold. She tells the nurse that he had "a runny nose for a week." When performing the physical assessment, the nurse notes that the child has nasal flaring and sternal and intercostal retractions. The nurse's next action should be to: A) assure the mother that these are normal symptoms of a cold. B) recognize that these are serious signs and contact the physician. C) ask the mother if the infant has had trouble with feedings. D) perform a complete cardiac assessment because these are probably signs of early heart failure.

ANS: B The infant is an obligatory nose breather until the age of 3 months. Normally there is no flaring of the nostrils and no sternal or intercostal retraction. Marked retractions of the sternum and intercostal muscles and nasal flaring indicate increased inspiratory effort, as in pneumonia, acute airway obstruction, asthma, and atelectasis; therefore, immediate referral to the physician is warranted. These signs do not indicate heart failure, and assessment of the infant's feeding is not a priority at this time.

When considering the biocultural differences in the respiratory systems, the nurse knows that which statement is true? A) The smallest chest volumes are found in Asians. B) The largest chest volumes are found in whites. C) Asians are most likely to develop asthma. D) Racial differences are of no significance when assessing the respiratory system.

ANS: B The largest chest volumes are found, in descending order, in whites, then African-Americans, Asians, and Native Americans. Even when the shorter height of Asians is considered, their chest volume remains significantly lower than that of whites and blacks. A disproportionately large number of tuberculosis cases are reported among blacks, most of whom were born in the United States.

During an examination of the anterior thorax, the nurse keeps in mind that the trachea bifurcates anteriorly at the: A) costal angle. B) sternal angle. C) xiphoid process. D) suprasternal notch.

ANS: B The sternal angle marks the site of tracheal bifurcation into the right and left main bronchi; it corresponds with the upper border of the atria of the heart, and it lies above the fourth thoracic vertebra on the back.

During an assessment of an adult, the nurse has noted unequal chest expansion and recognizes that this occurs in which situation? A) An obese patient B) When part of the lung is obstructed or collapsed C) When bulging of the intercostal spaces is present D) When accessory muscles are used to augment respiratory effort

ANS: B Unequal chest expansion occurs when part of the lung is obstructed or collapsed, as with pneumonia, or when guarding to avoid postoperative incisional pain or atelectasis.

A teenage patient comes to the emergency department with complaints of an inability to breathe and a sharp pain in the left side of his chest. The assessment findings include cyanosis, tachypnea, tracheal deviation to the right, decreased tactile fremitus on the left, hyperresonance on the left, and decreased breath sounds on the left. The nurse interprets that these assessment findings are consistent with: A) bronchitis. B) a pneumothorax. C) acute pneumonia. D) an asthmatic attack.

ANS: B With a pneumothorax, free air in the pleural space causes partial or complete lung collapse. If the pneumothorax is large, then tachypnea and cyanosis are seen. Unequal chest expansion, decreased or absent tactile fremitus, tracheal deviation to the unaffected side, decreased chest expansion, hyperresonant percussion tones, and decreased or absent breath sounds are found with the presence of pneumothorax. See Table 18-8 for descriptions of the other conditions.

When assessing the respiratory system of a 4-year-old child, which of these findings would the nurse expect? A) Crepitus palpated at the costochondral junctions B) No diaphragmatic excursion as a result of a child's decreased inspiratory volume C) The presence of bronchovesicular breath sounds in the peripheral lung fields D) An irregular respiratory pattern and a respiratory rate of 40 breaths per minute at rest

ANS: C Bronchovesicular breath sounds in the peripheral lung fields of the infant and young child up to age 5 or 6 years are a normal finding. Their thin chest walls with underdeveloped musculature do not dampen the sound, as do the thicker chest walls of adults, so breath sounds are louder and harsher.

The nurse is reviewing the characteristics of breath sounds. Which statement about bronchovesicular breath sounds is true? They are: A) musical in quality. B) usually pathological. C) expected near the major airways. D) similar to bronchial sounds except that they are shorter in duration.

ANS: C Bronchovesicular sounds are heard over major bronchi where fewer alveoli are located: posteriorly, between the scapulae, especially on the right; anteriorly, around the upper sternum in the first and second intercostal spaces. The other responses are not correct.

A woman in her 26th week of pregnancy states that she is "not really short of breath" but feels that she is aware of her breathing and the need to breathe. What is the nurse's best reply? A) "The diaphragm becomes fixed during pregnancy, making it difficult to take in a deep breath." B) "The increase in estrogen levels during pregnancy often causes a decrease in the diameter of the rib cage and makes it difficult to breathe." C) "What you are experiencing is normal. Some women may interpret this as shortness of breath, but it is a normal finding and nothing is wrong." D) "This is normal as the fetus grows because of the increased oxygen demand on the mother's body and results in an increased respiratory rate."

ANS: C During pregnancy, the woman may develop an increased awareness of the need to breathe. Some women may interpret this as dyspnea, even though structurally nothing is wrong. Estrogen increases relax the chest cage ligaments, causing an increase in transverse diameter. The growing fetus does increase the oxygen demand on the mother's body, but this is met easily by the increasing tidal volume (deeper breathing). Little change occurs in the respiratory rate.

A patient has been admitted to the emergency department with a possible medical diagnosis of pulmonary embolism. The nurse expects to see which assessment findings related to this condition? A) Absent or decreased breath sounds B) Productive cough with thin, frothy sputum C) Chest pain that is worse on deep inspiration, dyspnea D) Diffuse infiltrates with areas of dullness upon percussion

ANS: C Findings for pulmonary embolism include chest pain that is worse on deep inspiration, dyspnea, apprehension, anxiety, restlessness, PaO2 less than 80, diaphoresis, hypotension, crackles, and wheezes.

The nurse knows that auscultation of fine crackles would most likely be noticed in: A) a healthy 5-year-old child. B) a pregnant woman. C) the immediate newborn period. D) association with a pneumothorax.

ANS: C Fine crackles are commonly heard in the immediate newborn period as a result of the opening of the airways and clearing of fluid. Persistent fine crackles would be noticed with pneumonia, bronchiolitis, or atelectasis.

The nurse is reviewing the technique of palpating for tactile fremitus with a new graduate. Which statement by the graduate nurse reflects a correct understanding of tactile fremitus? "Tactile fremitus: A) is caused by moisture in the alveoli." B) indicates that there is air in the subcutaneous tissues." C) is caused by sounds generated from the larynx." D) reflects the blood flow through the pulmonary arteries."

ANS: C Fremitus is a palpable vibration. Sounds generated from the larynx are transmitted through patent bronchi and the lung parenchyma to the chest wall where they are felt as vibrations. Crepitus is the term for air in the subcutaneous tissues.

A patient has been admitted to the emergency department for a suspected drug overdose. His respirations are shallow, with an irregular pattern, with a rate of 12 per minute. The nurse interprets this respiration pattern as which of the following? A) Bradypnea B) Cheyne-Stokes respirations C) Hypoventilation D) Chronic obstructive breathing

ANS: C Hypoventilation is characterized by an irregular, shallow pattern, and can be caused by an overdose of narcotics or anesthetics. Bradypnea is slow breathing, with a rate less than 10 respirations per minute. See Table 18-4 for descriptions of Cheyne-Stokes respirations and chronic obstructive breathing.

During an assessment, the nurse knows that expected assessment findings in the normal adult lung include the presence of: A) adventitious sounds and limited chest expansion. B) increased tactile fremitus and dull percussion tones. C) muffled voice sounds and symmetrical tactile fremitus. D) absent voice sounds and hyperresonant percussion tones.

ANS: C Normal lung findings include symmetric chest expansion, resonant percussion tones, vesicular breath sounds over the peripheral lung fields, muffled voice sounds, and no adventitious sounds.

During auscultation of the lungs of an adult patient, the nurse notices the presence of bronchophony. The nurse should assess for signs of which condition? A) Airway obstruction B) Emphysema C) Pulmonary consolidation D) Asthma

ANS: C Pathologic conditions that increase lung density, such as pulmonary consolidation, will enhance transmission of voice sounds, such as bronchophony.

A 35-year-old recent immigrant is being seen in the clinic for complaints of a cough that is associated with rust-colored sputum, low-grade afternoon fevers, and night sweats for the past 2 months. The nurse's preliminary analysis, based on this history, is that this patient may be suffering from: A) bronchitis. B) pneumonia. C) tuberculosis. D) pulmonary edema.

ANS: C Sputum is not diagnostic alone, but some conditions have characteristic sputum production. Tuberculosis often produces rust-colored sputum in addition to other symptoms of night sweats and low-grade afternoon fevers.

The nurse is auscultating the chest in an adult. Which technique is correct? A) Instruct the patient to take deep, rapid breaths. B) Instruct the patient to breathe in and out through his or her nose. C) Use the diaphragm of the stethoscope held firmly against the chest. D) Use the bell of the stethoscope held lightly against the chest to avoid friction.

ANS: C The diaphragm of the stethoscope held firmly on the chest is the correct way to auscultate breath sounds. The patient should be instructed to breathe through his or her mouth, a little deeper than usual, but not to hyperventilate

A 65-year-old patient with a history of heart failure comes to the clinic with complaints of "being awakened from sleep with shortness of breath." Which action by the nurse is most appropriate? A) Obtain a detailed history of the patient's allergies and history of asthma. B) Tell the patient to sleep on his or her right side to facilitate ease of respirations. C) Assess for other signs and symptoms of paroxysmal nocturnal dyspnea. D) Assure the patient that this is normal and will probably resolve within the next week.

ANS: C The patient is experiencing paroxysmal nocturnal dyspnea: being awakened from sleep with shortness of breath and the need to be upright to achieve comfort.

The nurse notes hyperresonant percussion tones when percussing the thorax of an infant. The nurse's best action would be to: A) notify the physician. B) suspect a pneumothorax. C) consider this a normal finding. D) monitor the infant's respiratory rate and rhythm.

ANS: C The percussion note of hyperresonance occurs normally in the infant and young child, owing to the relatively thin chest wall. Anything less than hyperresonance would have the same clinical significance as would dullness in the adult.

When performing a respiratory assessment on a patient, the nurse notices a costal angle of approximately 90 degrees. This characteristic is: A) seen in patients with kyphosis. B) indicative of pectus excavatum. C) a normal finding in a healthy adult. D) an expected finding in a patient with a barrel chest.

ANS: C The right and left costal margins form an angle where they meet at the xiphoid process. Usually, this angle is 90 degrees or less. The angle increases when the rib cage is chronically overinflated, as in emphysema.

When auscultating the lungs of an adult patient, the nurse notes that over the posterior lower lobes low-pitched, soft breath sounds are heard, with inspiration being longer than expiration. The nurse interprets that these are: A) sounds normally auscultated over the trachea. B) bronchial breath sounds and are normal in that location. C) vesicular breath sounds and are normal in that location. D) bronchovesicular breath sounds and are normal in that location.

ANS: C Vesicular breath sounds are low-pitched, soft sounds with inspiration being longer than expiration. These breath sounds are expected over peripheral lung fields where air flows through smaller bronchioles and alveoli.

The nurse is performing an assessment on an adult. The adult's vital signs are normal and capillary refill time is 5 seconds. What should the nurse do next?

ANS: Consider this a delayed capillary refill time and investigate further. Normal capillary refill time is less than 1 to 2 seconds. The following conditions can skew the findings: a cool room, decreased body temperature, cigarette smoking, peripheral edema, and anemia.

When auscultating over a patient's femoral arteries the nurse notices the presence of a bruit on the left side. The nurse knows that:

ANS: bruits occur with turbulent blood flow, indicating partial occlusion. A bruit occurs with turbulent blood flow and indicates partial occlusion of the artery. The other responses are not correct.

During percussion, the nurse knows that a dull percussion note elicited over a lung lobe most likely results from: A) shallow breathing. B) normal lung tissue. C) decreased adipose tissue. D) increased density of lung tissue.

ANS: D A dull percussion note indicates an abnormal density in the lungs, as with pneumonia, pleural effusion, atelectasis, or tumor. Resonance is the expected finding in normal lung tissue.

A patient has a long history of chronic obstructive pulmonary disease. During the assessment, the nurse is most likely to observe which of these? A) Unequal chest expansion B) Increased tactile fremitus C) Atrophied neck and trapezius muscles D) An anteroposterior-to-transverse diameter ratio of 1:1

ANS: D An anteroposterior-to-transverse diameter of 1:1 or "barrel chest" is seen in individuals with chronic obstructive pulmonary disease because of hyperinflation of the lungs. The ribs are more horizontal, and the chest appears as if held in continuous inspiration. Neck muscles are hypertrophied from aiding in forced respiration. Chest expansion may be decreased but symmetric. Decreased tactile fremitus occurs from decreased transmission of vibrations.

The nurse is assessing the lungs of an older adult. Which of these describes normal changes in the respiratory system of the older adult? A) Severe dyspnea is experienced on exertion resulting from changes in the lungs. B) Respiratory muscle strength increases to compensate for a decreased vital capacity. C) There is a decrease in small airway closure, leading to problems with atelectasis. D) The lungs are less elastic and distensible, which decreases their ability to collapse and recoil.

ANS: D In the aging adult the lungs are less elastic and distensible, which decreases their ability to collapse and recoil. There is a decreased vital capacity and a loss of intraalveolar septa, causing less surface area for gas exchange. The lung bases become less ventilated, and the older person is at risk for dyspnea with exertion beyond his or her usual workload.

When inspecting the anterior chest of an adult, the nurse should include which assessment? A) Diaphragmatic excursion B) Symmetric chest expansion C) The presence of breath sounds D) The shape and configuration of the chest wall

ANS: D Inspection of the anterior chest includes shape and configuration of the chest wall; assessment of the patient's level of consciousness, skin color and condition; quality of respirations; presence or absence of retraction and bulging of the intercostal spaces; and use of accessory muscles. Symmetric chest expansion is assessed by palpation. Diaphragmatic excursion is assessed by percussion of the posterior chest. Breath sounds are assessed by auscultation.

During a morning assessment, the nurse notices that the patient's sputum is frothy and pink. Which condition could this finding indicate? A) Croup B) Tuberculosis C) Viral infection D) Pulmonary edema

ANS: D Sputum alone is not diagnostic, but some conditions have characteristic sputum production. Pink, frothy sputum indicates pulmonary edema (or it may be a side effect of sympathomimetic medications). Croup is associated with a "barking" cough, not sputum production. Tuberculosis may produce rust-colored sputum. Viral infections may produce white or clear mucoid sputum.

A 35-year-old man is seen in the clinic for an infection in his left foot. Which of these findings should the nurse expect to see during an assessment of this patient?

ANS: Enlarged and tender inguinal nodes The inguinal nodes in the groin drain most of the lymph of the lower extremities. With local inflammation, the nodes in that area become swollen and tender.

When performing an assessment of a patient, the nurse notices the presence of an enlarged right epitrochlear lymph node. What should the nurse do next?

ANS: Examine the patient's lower arm and hand, and check for the presence of infection or lesions. The epitrochlear nodes are located in the antecubital fossa and drain the hand and lower arm. The other actions are not correct for this assessment finding.

A patient is recovering from several hours of orthopedic surgery. During an assessment of the patient's lower legs, the nurse will monitor for signs of acute venous symptoms. Signs of acute venous symptoms include which of the following? Select all that apply.

ANS: Intense, sharp pain, with the deep muscle tender to touch Sudden onset Warm, red, and swollen calf Signs and symptoms of acute venous problems include pain in the calf that has a sudden onset and that is intense and sharp with tenderness in the deep muscle when touched. The calf is warm, red, and swollen. The other options are symptoms of chronic venous problems.

The nurse is reviewing venous blood flow patterns. Which of these statements best describes the mechanism(s) by which venous blood returns to the heart?

ANS: Intraluminal valves ensure unidirectional flow toward the heart. Blood moves through the veins by (1) contracting skeletal muscles that milk the blood proximally; (2) pressure gradients caused by breathing, in which inspiration makes the thoracic pressure decrease and the abdominal pressure increase; and (3) the intraluminal valves, which ensure unidirectional flow toward the heart.

The nurse is preparing to assess the dorsalis pedis artery. Where is the correct location for palpation?

ANS: Lateral to the extensor tendon of the great toe The dorsalis pedis artery is located on the dorsum of the foot. The nurse should palpate just lateral to and parallel with the extensor tendon of the big toe. The popliteal artery is palpated behind the knee. The posterior tibial pulse is palpated in the groove between the malleolus and the Achilles tendon. There is no pulse palpated at the lateral malleolus.

During an assessment, the nurse notices that a patient's left arm is swollen from the shoulder down to the fingers, with nonpitting brawny edema. The right arm is normal. The patient had a left-sided mastectomy 1 year ago. The nurse suspects which problem?

ANS: Lymphedema Lymphedema after breast cancer causes unilateral swelling and nonpitting brawny edema, with overlying skin indurated. It is caused by the removal of lymph nodes with breast surgery or damage to lymph nodes and channels with radiation therapy for breast cancer, and it can impede drainage of lymph. The other responses are not correct.

The nurse is reviewing an assessment of a patient's peripheral pulses and notices that the documentation states that the radial pulses are "2+." The nurse recognizes that this reading indicates what type of pulse?

ANS: Normal When documenting the force, or amplitude, of pulses, 3+ indicates an increased, full, or bounding pulse, 2+ indicates a normal pulse, 1+ indicates a weak pulse, and 0 indicates an absent pulse.

The nurse is performing a well-child check on a 5-year-old boy. He has no current history that would lead the nurse to suspect illness. His medical history is unremarkable, and he received immunizations 1 week ago. Which of these findings should be considered normal in this situation?

ANS: Palpable firm, small, shotty, mobile, nontender lymph nodes Palpable lymph nodes are often normal in children and infants. They are small, firm, shotty, mobile, and nontender. Vaccinations can produce lymphadenopathy. Enlarged, warm, tender nodes indicate current infection.

During an assessment of an older adult, the nurse should expect to notice which finding as a normal physiologic change associated with the aging process?

ANS: Peripheral blood vessels growing more rigid with age, producing a rise in systolic blood pressure Peripheral blood vessels grow more rigid with age, resulting in a rise in systolic blood pressure. Aging produces progressive enlargement of the intramuscular calf veins, not atrophy. The other options are not correct

The nurse is reviewing risk factors for venous disease. Which of these situations best describes a person at highest risk for development of venous disease?

ANS: Person who has been on bed rest for 4 days At risk for venous disease are people who undergo prolonged standing, sitting, or bed rest. Hypercoagulable (not anticoagulated) states and vein wall trauma also place the person at risk for venous disease. Obesity and pregnancy are also risk factors, but not the early months of pregnancy.

During an assessment, a patient tells the nurse that her fingers often change color when she goes out in cold weather. She describes these episodes as her fingers first turning white, then blue, then red with a burning, throbbing pain. The nurse suspects that she is experiencing:

ANS: Raynaud's disease. The condition with episodes of abrupt, progressive tricolor change of the fingers in response to cold, vibration, or stress is known as Raynaud's disease. Lymphedema is described in Table 20-2; deep vein thrombosis is described in Table 20-5; chronic arterial insufficiency is described in Table 20-4.

Which of these veins are responsible for most of the venous return in the arm?

ANS: Superficial The superficial veins of the arms are in the subcutaneous tissue and are responsible for most of the venous return.

When using a Doppler ultrasonic stethoscope, the nurse recognizes venous flow when which sound is heard?

ANS: Swishing, whooshing sound When using the Doppler ultrasonic stethoscope, the pulse site is found when one hears a swishing, whooshing sound.

Which of these statements is true regarding the arterial system?

ANS: The arterial system is a high-pressure system. The pumping heart makes the arterial system a high-pressure system.

A patient has been admitted with chronic arterial symptoms. During the assessment, the nurse should expect which findings? Select all that apply.

ANS: The patient has a history of diabetes and cigarette smoking. The patient's skin is pale and cool. The patient states that the pain gets worse when walking. See Table 20-3. Patients with chronic arterial symptoms often have a history of smoking and diabetes (among other risk factors). The pain has a gradual onset, with exertion, and is relieved with rest or dangling. The skin appears cool and pale. The other responses reflect chronic venous problems.

The nurse is examining the lymphatic system of a healthy 3-year-old child. Which finding should the nurse expect?

ANS: The presence of palpable lymph nodes Lymph nodes are relatively large in children, and the superficial ones often are palpable even when the child is healthy.

The nurse is assessing a patient's pulses and notices a difference between the patient's apical pulse and radial pulse. The apical pulse was 118 beats per minute; the radial pulse was 105 beats per minute. Calculate the pulse deficit. Pulse deficit equals: _________

ANS: The pulse deficit equals 13 beats per minute.

The nurse is preparing to perform a modified Allen test. Which is an appropriate reason for this test?

ANS: To evaluate the adequacy of collateral circulation before cannulating the radial artery A modified Allen test is used to evaluate the adequacy of collateral circulation before the radial artery is cannulated. The other responses are not reasons for a modified Allen test.

During a clinic visit, a woman in her seventh month of pregnancy complains that her legs feel "heavy in the calf" and that she often has foot cramps at night. The nurse notices that the patient has dilated, tortuous veins in her lower legs. Which condition is reflected by these findings?

ANS: Varicose veins Superficial varicose veins are caused by incompetent distant valves on veins, which results in reflux of blood and producing dilated, tortuous veins. They are more common in women, and pregnancy can also be a cause. Symptoms include aching, heaviness in the calf, easy fatigability, and night leg or foot cramps. Dilated, tortuous veins are seen on assessment. See Table 20-5 for the description of deep vein thrombophlebitis. See Table 20-2 for descriptions of Raynaud's phenomenon and lymphedema.

During a routine office visit, a patient takes off his shoes and shows the nurse "this awful sore that won't heal." On inspection, the nurse notes a 3-cm round ulcer on the left great toe, with a pale ischemic base, well-defined edges, and no drainage. The nurse should assess for other signs and symptoms of:

ANS: an arterial ischemic ulcer. Arterial ischemic ulcers occur at toes, metatarsal heads, heels, and lateral ankle, and they are characterized by a pale ischemic base, well-defined edges, and no bleeding. See Table 20-5 for a description of varicose veins and deep vein thrombophlebitis. See Table 20-4 for a description of venous stasis ulcers.

A 67-year-old patient states that he recently began to have pain in his left calf when climbing the 10 stairs to his apartment. This pain is relieved by sitting for about 2 minutes; then he is able to resume his activities. The nurse interprets that this patient is most likely experiencing:

ANS: claudication. Intermittent claudication feels like a "cramp" and is usually relieved by rest within 2 minutes. The other responses are not correct.

When performing a peripheral vascular assessment on a patient, the nurse is unable to palpate the ulnar pulses. The patient's skin is warm and capillary refill time is normal. The nurse should next:

ANS: consider this a normal finding and proceed with the peripheral vascular evaluation. It is not usually necessary to palpate the ulnar pulses. The ulnar pulses are often not palpable in the normal person. The other responses are not correct.

A patient has a positive Homans' sign. The nurse knows that a positive Homans' sign may indicate:

ANS: deep vein thrombosis. Calf pain on dorsiflexion of the foot is a positive Homans' sign, which occurs in about 35% of deep vein thromboses. It also occurs with superficial phlebitis, Achilles tendinitis, and gastrocnemius and plantar muscle injury.

During an assessment, the nurse uses the "profile sign" to detect:

ANS: early clubbing. The nurse should use the profile sign (viewing the finger from the side) to detect early clubbing.

A 65-year-old patient is experiencing pain in his left calf when he exercises that disappears after resting for a few minutes. The nurse recognizes that this description is most consistent with _________ the left leg.

ANS: ischemia caused by partial blockage of an artery supplying Ischemia is a deficient supply of oxygenated arterial blood to a tissue. A partial blockage creates an insufficient supply, and the ischemia may be apparent only during exercise when oxygen needs increase.

A patient has hard, nonpitting edema of the left lower leg and ankle. The right leg has no edema. Based on these findings, the nurse recalls that:

ANS: nonpitting, hard edema occurs with lymphatic obstruction Unilateral edema occurs with occlusion of a deep vein and with unilateral lymphatic obstruction. With these factors, the edema is nonpitting and feels hard to the touch (brawny edema).

When assessing a patient's pulse, the nurse notes that the amplitude is weaker during inspiration and stronger during expiration. When the nurse measures the blood pressure, the reading decreases 20 mm Hg during inspiration and increases with expiration. This patient is experiencing pulsus:

ANS: paradoxus. In pulsus paradoxus, beats have a weaker amplitude with inspiration and a stronger amplitude with expiration. It is best determined during blood pressure measurement; reading decreases (>10 mm Hg) during inspiration and increases with expiration.

A patient complains of leg pain that wakes him at night. He states that he "has been having problems" with his legs. He has pain in his legs when they are elevated that disappears when he dangles them. He recently noticed "a sore" on the inner aspect of the right ankle. On the basis of this history information, the nurse interprets that the patient is most likely experiencing:

ANS: problems related to arterial insufficiency. Night leg pain is common in aging adults. It may indicate the ischemic rest pain of peripheral vascular disease. Alterations in arterial circulation cause pain that becomes worse with leg elevation and is eased when the extremity is dangled.

The nurse is performing a peripheral vascular assessment on a bedridden patient and notices the following findings in the right leg: increased warmth, swelling, redness, tenderness to palpation, and a positive Homan's sign. The nurse should:

ANS: seek emergency referral because of the risk of pulmonary embolism. Increased warmth, swelling, redness, and tenderness in the lower extremities require emergency referral because of the risk of pulmonary embolism from a deep vein thrombosis.

A nurse palpates a client's hands and fingers. Which of the following findings would be consistent with arterial insufficiency?

cool skin

During an assessment, the nurse first performs the action shown. (legs up) After that the nurse asks the client to sit up with their legs dangling from the edge of the table. What is the nurse assessing?

The color change test is to check for arterial insufficiency. With the patient supine, the legs are elevated about 30 cm (12 in.) above the level of the heart. Then when have the patient sit up and dangle the legs. Color should return to the feet and toes within 10 seconds. The superficial veins of the feet fill within 15 seconds. Return of color taking longer than 10 seconds or persistent dependent rubor indicates arterial insufficiency. This is not a technique to assess lymphedema, the femoral pulse, or intermittent claudication.

The nurse is instructing a client about high cholesterol level and wants to include behavioral considerations. What should the nurse include in this instruction? Hint: Focused Interview Questions; Questions Related to Behaviors The need for an annual cholesterol panel The need for annual flu inoculation The need to stop smoking The need to reduce stress

The need to stop smoking Behavioral considerations to include in this instruction include smoking cessation, diet, and the need for regular exercise. Behavioral considerations include factors that the client can modify. The client may need an annual cholesterol panel, but it is not related to behavioral modification. Influenza immunization is not related to high cholesterol. Stress may be related to the onset of chest pain but not elevated cholesterol.

What pulse is located in the groove between the medial malleolus and the Achilles tendon?

The posterior tibial pulse is located in the groove between the medial malleolus and the Achilles tendon. The femoral pulse is about halfway between the symphysis pubis and the anterior iliac spine, just below the inguinal ligament. The popliteal pulse is often difficult to locate. It may be felt immediately lateral to the medial tendon. A light touch is important to avoid obliterating the dorsalis pedis pulse. It is normally about halfway up the foot immediately lateral to the extensor tendon of the great toe.

During cardiac auscultation, the nurse hears a split S2 heart sound. Which is true regarding this physiologic sound? Hint: Anatomy and Physiology Review, Heart Sounds The aortic valve is closing slightly before the tricuspid valve. The pulmonic valve is closing slightly after the aortic valve. This is an abnormal finding. The mitral valve closes slightly before the tricuspid valve.

The pulmonic valve is closing slightly after the aortic valve. A split S2 is heard during inspiration in many individuals. It is the result of the semilunar valves closing at slightly different times. The aortic valve closes slightly faster than the pulmonic valve due to changes in intrathoracic pressure caused by inspiration. A split S2 during inspiration is a normal (physiologic) sound and doesn't signify cardiac pathology. The mitral valve closes slightly before the tricuspid valve and is responsible for the S1 heart sound, not a split S2.

A young man is concerned about a hard mass in the midline of his palate that he has just noticed. Examination reveals that it is indeed hard and in the midline. No mucosal abnormalities are associated with this lesion. The client has no other symptoms. What is the most likely diagnosis?

Torus palatinus Torus palatinus is relatively common and benign but can go unnoticed by clients for many years. The appearance of a bony mass can be concerning. Leukoplakia is a white lesion on the mucosal surfaces corresponding to chronic mechanical or chemical irritation. It can be premalignant. Thrush is usually painful and seen in immunosuppressed clients or those taking inhaled steroids for COPD or asthma. Kaposi's sarcoma is usually seen in HIV-positive people; these lesions are classically deep purple.

A nurse is inspecting a client's nipples. Which of the following findings should the nurse regard as a cause for concern?

a recently retracted nipple that was previously everted A recently retracted nipple that was previously everted suggests malignancy. Normal findings include nipples being nearly equal in size and in the same location on each breast. Nipples are usually everted, but they may be inverted or flat. Supernumerary nipples may appear along the embryonic "milk line."

The nurse identifies a heart sound heard during the interval between the first heart sound (S1) and the second sound (S2) as: a systolic sound a diastolic sound an atrial filling sound a ventricular contraction sound

a systolic sound The period between S1 and S2 is systole. Any sound heard after S1 and before S2 would be called a systolic sound. The period between S2 and the next S1 is diastole. Any sound heard during this would be called a diastolic sound. Filling of the atria does not produce a sound. The ventricles do not produce a sound when contracting.

A client reports experiencing chronic headache after a recent upper respiratory tract infection. On physical examination, the nurse notes tenderness when palpating over the sinuses. Which condition is likely?

acute bacterial sinusitis Acute bacterial sinusitis should be suspect if there is a recent history of upper respiratory tract infection. The sensitivity for the symptom correlation of upper respiratory tract infections is 90%, and the specificity is 80%. Headache and tenderness on palpation of the sinuses are also features acute bacterial sinusitis. Allergic rhinitis is not associated with an infectious process. Nasal congestion that results from triggering allergens, usually environmental, is the cause. Rhinitis medicamentosa results from excessive use of decongestants used to treat a nonbacterial nasal congestion. Epistaxis is a condition that occurs when the mucosa of the nares is eroded and exposed vessels break leading to what is commonly called a nosebleed. Tenderness and headache are not associated features of epistaxis.

What is a long-term complication of peripheral vascular disease?

amputation Diseases of the peripheral vascular system, peripheral arterial disease, venous stasis, and thromboembolic disorders can severely affect the lifestyle and quality of life of patients. Identifying modifiable risk factors and providing health promotion counseling can prevent or delay long-term complications, such as decreased mobility and amputation.

While assessing the peripheral vascular system of an adult client, the nurse detects cold clammy skin and loss of hair on the client's legs. The nurse suspects that the client may be experiencing

arterial insufficiency Manifestations of arterial insufficiency include intermittent claudication to sharp, unrelenting, and constant. Diminished or absent pulses. Skin in cool to cold in temperature and there is a loss of hair over the toes and dorsum of the foot.

A nurse cares for a client who is postoperative cholecystectomy. Which action by the nurse is appropriate to help prevent the occurrence of venous stasis?

assist the client to walk as soon and as often as possible Immobility creates an environment in which clotting (embolism formation) can be caused by venous stasis. Active exercise such as having the client ambulate as soon as possible will stimulate circulation and venous return. This reduces the possibility of clot formation. Raising the foot of the bed, vigorous massage, and active range of motion of the upper body may not prevent venous stasis.

Screening measures for breast cancer include all of the following except:

breast x rays

The nurse documents a 2+ radial pulse. What assessment data indicated this result?

brisk, expected (normal) pulse

Which of the following is an essential topic when discussing risk factors for peripheral arterial disease with a client?

extent of tobacco use and exposure

The client has been diagnosed with peripheral arterial disease. What information should the nurse include when teaching?

check feet daily for cuts and pressure areas Because of decreased blood flow, the client needs to check feet daily for cuts or pressure areas so that treatment to prevent arterial ulcers can begin immediately. Compression stockings should not be worn by clients with peripheral arterial disease. Disfigurement is not common in clients with peripheral arterial disease. There will be decreased hair and the skin will be shiny in clients with peripheral arterial disease.

A client with a right subclavian central line develops fever of 101.0 degrees Fahrenheit. What is the nurse's best action?

check the insertion site for redness Fever above 100.4 degrees Fahrenheit can indicate a central-line associated bloodstream infection for this client. The nurse should assess the insertion site for redness, edema, or purulent drainage and notify the healthcare provider for further treatment. Depending on the signs of infection that are present at the insertion site, the provider may discontinue the line and culture the tip. Flushing the ports with saline can assist the nurse in checking patency of the lines.

A client presents to the health care clinic with reports of swelling, pain, and coolness of the lower extremities. The nurse should recognize that which of these lifestyle practices are risk factors for peripheral vascular disease? Select all that apply.

cigarette smoking previous use of hormones high fat diet

Which type of discharge should a nurse consider normal with palpation of the nipples during a breast examination of a non-lactating female client?

clear A clear discharge is a normal finding that may be obtained on manual expression from a breast that is frequently stimulated. Milky discharge, blood-tinged discharge, or discharge from one breast are abnormal findings in this examination. A milky discharge is normal only during pregnancy or lactation. A client with any blood or blood-tinged discharge should be referred to a physician for further evaluation. Discharge from one breast may indicate benign intraductal papilloma, fibrocystic disease, or cancer of the breast.

Which of the following assessment findings is most congruent with chronic arterial insufficiency?

cool temp and ulceration on clients great toe Pigmentation, medial ankle ulceration, and thickened, scarred skin are associated with venous insufficiency, while low temperature and toe ulceration are more commonly found in cases of arterial insufficiency.

The diagnosis of superficial phlebitis increases the client's risk for which vascular disorder?

deep vein thrombosis Superficial phlebitis is an inflammation of a superficial vein that can lead to deep vein thrombosis. Compartment syndrome is a result of pressure building from trauma or bleeding into one of the four major muscle compartments between the knee and ankle. Acute lymphangitis is a bacterial infection from Streptococcus pyogenes or Staphlococcus aureus, spreading up the lymphatic channels from a distal portal of entry. Acute cellulitis is a bacterial infection of the skin and subcutaneous tissues.

A client has had a recent mastectomy and visits the clinic for postoperative evaluation. The client tells the nurse that she has been depressed and feels as if she is less of a woman. The most appropriate nursing diagnosis for this client is

disturbed body image related to mastectomy When interviewing clients—especially females—about the breasts, keep in mind that this topic may evoke a wide spectrum of emotions from the client. Explore your own feelings regarding body image, fear of breast cancer, and the influence of the breasts on self-esteem. Disturbed body image is the most appropriate nursing diagnosis.

The size and shape of the breasts in females are related to the amount of

fatty tissue Fatty tissue is the third component of the breast. The glandular tissue is embedded in the fatty tissue. This subcutaneous and retromammary fat provides most of the substance to the breast, determining the size and shape of the breasts.

The nurse is assessing a 15 year old male and finds soft, fatty enlargement of breast tissue. The nurse would document this as what?

gynecomastia Gynecomastia is breast enlargment. Cysts are lumps that may be found in the breasts. Abscesses are an infection. Fibroadenoma s a well-defi ned, usually single or multiple, nontender, fi rm or rubbery, round or lobular mass that is freely movable.

The nurse has discussed the risks for breast cancer with a group of high school seniors. The nurse determines that one of the students needs further instructions when the student says that one risk factor is

having a baby before the age of 20 years The risk of breast cancer is greater for women who have never given birth or for those who had their first child after age 30.

While interviewing a client, the nurse asks her what her typical daily diet consists of. Which of the following is associated with an increased risk for breast cancer?

high fat diet A high-fat diet may increase the risk for breast cancer. Alcohol intake exceeding two drinks per day and tobacco use has been associated with a higher risk for breast cancer. Caffeine can aggravate fibrocystic breast disease, but is not associated with breast cancer. A high-sugar diet is not associated with breast cancer.

Which nursing assessment findings support a medical diagnosis of acute lymphangitis? Select all that apply.

history of animal bite red streak noted on skin fever is present Acute lymphangitis presents with red streak(s) on the skin, with tenderness, enlarged, tender lymph nodes, and fever. Bacteria is often introduced by a animal bite. Compartment syndrome presents with pressure and numbness.

A client describes a 3-week history of hoarseness. The client also reports feeling fatigue and noticeable weight gain over the past month. Which cause should the nurse consider as most likely?

hypothyroidism Hoarseness lasting longer than two weeks accompanied by the additional reported symptoms of fatigue and weight gain suggest hypothyroidism. Gingivitis is an inflammation of the gums that often results in bleeding. Dysphagia is a medical term used to describe difficulty swallowing. This can be due to a neuromuscular or oropharyngeal deficit. Hoarseness can accompany dysphagia; however, weight gain and fatigue do not. Apthous ulcers are a common condition that are restricted to the oral cavity. Commonly called "canker sores," they may be associated with autoimmune problems that create a predisposition to developing oral ulcers.

A client presents to the health care clinic with a 3-week history of pain and swelling of the right foot. A nurse inspects the foot and observes swelling and a large ulcer on the heel. The client reports the right heel is very painful and he has trouble walking. Which nursing diagnosis should the nurse confirm from these data?

impaired skin integrity This client demonstrates Impaired Skin Integrity as evidenced by the ulcer on his heel. With the location and the presence of pain, this is most likely to be an ulcer of arterial insufficiency. The client has not verbalized any fear at this time. With the existing skin breakdown, he is not at risk because it is present. No nutritional imbalances are documented.

The nurse is preparing to examine the breasts of a female client who had a left radical mastectomy 3 years ago. When examining the client, the nurse observes redness at the scar area. The nurse should explain to the client that this may be indicative of

infectious problem Redness and inflammation of the scar area may indicate infection.

The nurse palpates a mobile mass that becomes fixed when the client's arm relaxes. What would this tell the nurse about the mass?

it is attached to the ribs and intercostal muscles A mobile mass that becomes fixed when the arm relaxes is attached to the ribs and intercostal muscles; if fixed when the hand is pressed against the hip, it is attached to the pectoral fascia

A nurse performs the Trendelenburg test for a client with varicose veins. Which action should the nurse take when performing this test?

legs should be elevated for 15 seconds When performing the Trendelenburg test, the nurse should elevate the client's leg for 15 seconds to empty the veins. The tourniquet should be put on after leg elevation. The client should stand upright with the tourniquet on the leg. The client is not asked to sit with the leg hanging down when performing the Trendelenburg test.

A 43-year-old store clerk comes to the office upset because she has found an enlarged lymph node under her left arm. She states she found it yesterday when she was feeling pain under her arm during movement. She states the lymph node is about an inch long and is very painful. She checks her breasts monthly and gets a yearly mammogram (her last was 2 months ago); until now everything has been normal. She states she is so upset because her mother died in her 50s of breast cancer. The client does not smoke, drink, or use illegal drugs. Her father is in good health. Examination shows a tense woman appearing her stated age. Visual inspection of her left axilla reveals a tense red area with no surrounding scarring. On palpation, the examiner feels a 2-cm tender movable lymph node underlying hot skin. Other shoddy nodes are also in the area. Visualization of both breasts is normal. Palpation of her right axilla and both breasts is unremarkable. Examination of the left arm reveals a scabbed-over superficial laceration over her left hand. Upon questioning, the client remembers that she cut her hand gardening last week. What disorder of the axilla is most likely responsible for her symptoms?

lymphadenopathy of infectious origin

Your patient is noted to have thickening of the skin and unusually prominent pores on her right breast. What might you suspect?

lymphatic obstruction Thickening of the skin and unusually prominent pores may accompany lymphatic obstruction.

During the physical examination of a female client, the nurse notes that the client's axillary lymph nodes are enlarged, hard, and fixed. The nurse recognizes that these findings are consistent with what disease process?

malignancy Enlarged, hard, and fixed axillary lymph nodes indicate malignancy. Infection may cause swelling of the lymph nodes but does make them hard and fixed. Lactation does not cause a change in lymph nodes. Inflammation does not cause the lymph nodes to harden and get fixed.

During the cardiac assessment, the nurse learns that a client had rheumatic fever as a child. The nurse should carefully assess this client for: Hint: Valvular Disease, Mitral Stenosis chest pain mitral valve prolapse mitral stenosis pulmonic stenosis

mitral stenosis Mitral stenosis, the narrowing of the mitral valve, can be caused by rheumatic fever or another cardiac infection. Mitral stenosis produces a diastolic murmur heard best at the apex of the heart. Rheumatic fever does not cause chest pain. Mitral valve prolapse has unknown etiology. Pulmonic stenosis is congenital in origin and not related to rheumatic fever.

If palpable, superficial inguinal nodes are expected to be:

nontender, mobile, 1 cm in diameter

A nurse assists the client to perform the position change test for arterial insufficiency. While dangling the legs, the nurse observes a return of color to the feet in 8 seconds. How should the nurse document the finding for this test?

normal Return of a pink color to the legs after elevation should take less than 10 seconds. This test does not demonstrate arterial insufficiency. Delayed would be greater than 10 seconds for color to return.

When assessing the lymph system of an adult client, the nurse notes that the epitrochlear nodes are nonpalpable. What does this indicate?

normal finding Normally, the epitrochlear nodes are not palpable. Normal palpable nodes are 2 cm or less. Nonpalpable epitrochlear nodes are not an indication of lymphoma or atherosclerosis. They are not related to lymphedema or its absence.

A hospitalized post-operative client exhibits edema, pain, erythema, and warmth in the right calf area. What is the nurse's best action?

notify the healthcare provider The client is exhibiting signs of venous thromboembolism. The healthcare provider should be notified immediately to prevent further complications. This condition is a national client safety concern for hospitalized clients. Early ambulation could dislodge a possible clot. Prevention of pneumonia is encouraged by turning, coughing, and deep breathing. Signs of a urinary tract infection include pain, increased white blood cells, and fever.

A nurse recognizes that a common complication of vascular surgery may manifest as which assessment finding?

pain in the calf muscles Clients undergoing vascular surgery are at increased risk for the development of deep vein thrombosis. The Homan's test has traditionally been used to detect the presence of a blood clot within a vessel. Homan's sign is positive if the client experiences tenderness or pain in the calf muscles on flexing the knee, and aching or cramping on dorsiflexion of the foot. Cramping pain in thighs may not be elicited by Homan's sign. Pallor of the leg on elevation is not elicited by Homan's test. Tenderness on plantar flexion of foot indicates negative Homan's sign.

The nurse understands that the pericardial space is formed by the: Hint: Anatomy and Physiology Review, Pericardium parietal and visceral layers myocardium and the endocardium pericardium and the mediastinal space myocardium and the epicardium

parietal and visceral layers The pericardial space is between the parietal and visceral pericardial layers. The parietal layer is the outer layer. The visceral layer is the inner layer, which lines the surface of the heart. This is the location where infection or inflammation can occur and lead to pericarditis. The myocardium is the thick, muscular layer of the heart, and the innermost layer is the endocardium. The pericardium surrounds the heart. The area between the lungs and above the diaphragm is called the mediastinal space.

A client complains of pain in the calves, thighs, and buttocks whenever he climbs more than a flight of stairs. This pain, however, is quickly relieved as soon as he sits down and rests. The nurse should suspect which of the following conditions in this client?

peripheral artery disease Intermittent claudication is characterized by weakness, cramping, aching, fatigue, or frank pain located in the calves, thighs, or buttocks but rarely in the feet with activity. These symptoms are quickly relieved by rest but reproducible with same degree of exercise and may indicate peripheral arterial disease (PAD). Leg pain that awakens a client from sleep is often associated with advanced chronic arterial occlusive disease. A lack of pain sensation may signal neuropathy in such disorders as diabetes. Heaviness and an aching sensation aggravated by standing or sitting for long periods of time and relieved by rest are associated with venous disease.

A client reports pain in the legs that begins with walking but is relieved by rest. Which condition should the nurse assess the client for?

peripheral vascular problems The nurse should assess the client for peripheral vascular problems in both the legs. Intermittent claudication is a condition that indicates vascular deficiencies in the peripheral vascular system. In case of an acute obstruction, the leg pain would persist even when the client stopped walking. Diabetes can cause pain as a result of diabetic neuropathy, which is unrelated to walking. Low calcium level may cause leg cramps but would not necessarily be related to walking.

After assessing pitting edema below the knee in a patient, the nurse would suspect that which vein may be occluded?

popliteal Although normal popliteal arteries may be nonpalpable, an absent pulse may also be the result of an occluded artery. Further circulatory assessment such as temperature changes, skin-color differences, edema, hair distribution variations, and dependent rubor (dusky redness) distal to the popliteal artery assists in determining the significance of an absent pulse.

The nurse plans to instruct an adult female client with regular menstrual cycles, who is not taking oral contraceptives, about breast self-examination. The nurse should plan to instruct the client to perform breast self-examination

right after menstruation

The nurse is assessing a client's lymphatic system. For which enlarged node should the nurse suspect that the client has a blockage within the right lymphatic duct?

right cervical node The right cervical node drains into the right lymphatic duct. The lumbar, superficial inguinal, and superficial popliteal nodes drain into the thoracic duct.

When palpating the female breast for masses, the nurse distinguishes which of the following characteristics as a potentially cancerous mass?

single, firm, fixed nodule

While performing a routine check-up on an 81-year-old retired grain farmer in the vascular surgery clinic, the nurse notes that he has a history of chronic arterial insufficiency. Which of the following physical examination findings of the lower extremities would be expected with this disease?

thin shiny atrophic skin

An adult client visits the clinic complaining of a sore throat. After assessing the throat, the nurse documents the client's tonsils as 4+. The nurse should explain to the client that 4+ tonsils are present when the nurse observes tonsils that are

touching each other.

The radial pulse is palpated over the lateral flexor surface.

true

A nurse is examining the breasts of a client. In which quadrant should the nurse most expect to find a tumor?

upper outer quadrant


Ensembles d'études connexes

Chapter 1 Business Finance Smart Book

View Set

AP Gov Unit 5 - College Board Review

View Set

Practice MCQ Questions from FINAL REVIEW + LECTURES

View Set

Coltman - Environmental Biology Test 3

View Set

A+ Guide to Managing Ch. 3 Review Questions

View Set

Chapter 14: Main DSM-5 Categories of Mental Disorders

View Set